LEGIT Psychiatry Cases 2

Réussis tes devoirs et examens dès maintenant avec Quizwiz!

A 70-year-old woman presents to her primary care provider complaining of fatigue for the past 7 weeks. She admits to difficulty falling asleep, a poor appetite with a 10-lb weight loss, and thoughts of wanting to die. She admits to having had symptoms similar to these on several occasions in the past, but "never this bad." Her medical problems include asthma and high cholesterol. She uses an albuterol inhaler only as needed. Which of the following symptoms is necessary in order to make a diagnosis of major depressive disorder? A. Depressed mood B. Decreased appetite C. Excessive guilt D. Fatigue E. Suicidal ideation

A. Although a change in appetite, decreased energy, fatigue, and suicidal ide- ation are all criteria used in diagnosing major depressive disorder, one of the symptoms must be either a depressed mood or anhedonia.

In the previous case, the patient is determined to have a delirium due to infec- tion, overlying his dementia. Which of the following is the most important treatment approach in treating his delirium? A. Detection and correction of the underlying abnormality B. Environmental strategies to help with orientation C. Treatment with an antipsychotic medication for hallucinations D. Physical restraint to protect the patient from injury E. Treatment with a benzodiazepine to reduce agitation

A. Although environmental strategies and pharmacologic and physical inter- ventions can be helpful and necessary to help orient patients or protect them from harm, the most essential treatment approach in all cases of delirium is to detect and correct the underlying cause of the disorder. The occurrence of an episode of delirium itself suggests a poor prognosis, meaning these patients have a significantly elevated future incidence of mortality. In the elderly, antipsychotic agents should be used with caution given some increased risk of mortality, and benzodiazepines may cause disinhibition, oversedation, or paradoxical excitation.

A 9-year-old boy is referred to a psychiatrist because of poor school perfor- mance. He has been tested for learning disabilities but none are present, with an IQ in the high normal range. The teacher reports that it is hard to hold his attention. In addition, he appears hyperactive and fidgety at school, which dis- rupts the class. However, he does not purposefully go out of his way to disobey the teacher. His parents have noticed no difficulties at home, but his soccer coach has noticed attention problems during practice, and his Sunday school teacher has trouble teaching him because of distractibility. Which of the fol- lowing is the most likely diagnosis for this patient? A. ADHD, combined type B. ADHD, predominantly hyperactive type C. ADHD, predominantly inattentive type D. Oppositional defiant disorder E. No diagnosis, because the ADHD symptoms must be reported in the home

A. Attention-deficit disorder, combined type. The diagnostic criteria for ADHD require that the symptoms be present in more than one setting, usu- ally at home and at school. However, this child seems to have evidence of symptoms observed at school, church, and soccer. The child had prominent distractibility and hyperactivity.

Which of the following is the treatment of choice for social phobia? A. Behavior therapy B. SSRIs C. Electroconvulsive therapy D. Psychoanalysis E. Divalproex sodium (Depakote)

A. Behavior therapy is the treatment of choice for social phobia. Benzodiaz- epines can be used to reduce the associated anxiety. A beta-blocker such as propranolol can also help reduce the autonomic hyperarousal that occurs in social settings. In addition, some SSRIs are also useful.

An 82-year-old man with a history of vascular dementia is brought to the hos- pital for increased agitation and urinary tract infection (UTI). Which of the following features most distinguishes effects of a delirium from dementia? A. Altered level of consciousness B. Behavioral disturbances C. Cognitive deficits D. Disorientation E. Presence of hallucinations

A. Both delirium and dementia can result in behavioral disturbances, cogni- tive deficits, and poor orientation. However, in all cases of delirium there is an alteration (reduction) in the level of consciousness, whereas in dementia (in the early stages) there is an alert, stable level of consciousness.

A 27-year-old woman and her 7-year-old son present to a mental health center for treatment. The patients were passengers in the back of the family car, when they were struck by a semitractor trailer, which killed the father and an older sister. Both mother and son endorse significant depressive symptoms. Which of the following symptoms would most likely differ between the presentations of these two patients? A. Irritability B. Suicidal thoughts C. Flashbacks D. Insomnia E. Inattention

A. In the clinical presentation of children and adolescents, one will often find evidence of irritability or short temper rather than a feeling of sadness or depression. The ability to understand the concept of depression seems to be developmentally mediated.

A 24-year-old man presents to a therapist. Which of the following statements made by the patient is most consistent with avoidant personality disorder? A. "I have a couple of close friends, but it seems like I just don't get most people." B. "I'm usually fine around people. It's just when I'm around a lot of people I've never met before that I get freaked out." C. "I'm afraid that people are plotting against me." D. "My mom thinks I have a problem with people. I can take them or leave them." E. "My girlfriend thinks I have a problem with people, like with her friends. What do you think?"

A. Option A is most consistent with avoidant personality disorder. The patient has some close relationships and seems to desire more, but does not feel that he can maintain such relationships. B is more consistent with social phobia. C is grossly paranoid, consistent with a psychotic disorder. D is highly suggestive of schizoid personality disorder, as the patient has a close relationship with his parent but has no interest in other human contact. E suggests dependent personality disorder.

A 36-year-old man with narcissistic personality disorder calls your office asking for an appointment with the "best therapist in the clinic." One of his com- plaints is difficulties in his relationships with his colleagues. The patient states "They are not giving him the credit he deserves for his accomplishments at the law firm." What is the most likely reason the patient is seeking treatment? A. Anger B. Anxiety C. Attempting to identify with others D. Grandiose thinking E. Seeking medication

A. Patients with narcissistic personality disorder rarely seek treatment and tend to have little insight into their grandiosity. When these individuals do present for treatment it is usually due to underlying anger or depression resulting from being belittled or not receiving the admiration to which they feel entitled.

Social phobia differs from specific phobia in which of the following ways? A. Focus or nature of the fear B. Duration of the illness C. Absence of panic attacks D. Degree of avoidance of the situation E. Recurrent nature of the fear

A. Social phobia might be also considered a specific phobia under some cir- cumstances and is very similar in duration, progression, symptoms, and patient avoidance. The real difference is the nature of the fear—in social phobia this is characterized by social situations.

A 52-year-old man presents to his primary care physician after the death of his wife from breast cancer 2 months ago. He complains of depression, inconsolable sadness, frequent crying, and an inability to focus upon his work and usual activi- ties. Which of the following treatments would likely be most helpful for him? A. Supportive psychotherapy B. Family therapy C. A selective serotonin reuptake inhibitor antidepressant D. Psychoanalysis E. Behavioral modification therapy

A. Supportive psychotherapy is indicated to help the patient deal with his response to his loss, either in an individual or a group setting. Medications are not indicated for bereavement, except perhaps a mild sleep aid if insomnia is a problem. Behavioral modification and/or psychoanalysis are both unnecessary in this setting, since a much more acute problem which is not behavioral in nature is at issue. The patient does not indicate familial problems (other than his wife's death) so supportive therapy is the best option.

A 78-year-old man presents with 4 weeks of significant depression following the sudden, unforeseen death of his wife of 35 years. He reports difficulty sleeping, a 10-lb weight loss, frequent crying spells, and profound guilt over surviving her. For the last several days, he has been convinced that his body is literally decaying. He admits to seeing his wife's face during the day, as well as hearing her voice telling him to kill himself and join her. A. Major depression with psychotic features B. Schizoaffective disorder C. Schizophrenia D. Psychosis secondary to a general medical condition E. Substance-induced psychotic disorder

A. The most likely diagnosis for this man is major depression with psychotic features. Significant depression and neurovegetative symptoms are present, as well as delusions and auditory and visual hallucinations. Although he has mood symptoms and psychotic symptoms, his history is consistent with major depres- sion because his mood symptoms preceded his psychotic symptoms.

A 28-year-old male assembly-line worker comes in for treatment after devel- oping symptoms a few days after a serious factory accident in which he was knocked unconscious by a machine. He was medically cleared directly follow- ing the event but later developed nightmares about the accident. He says he has been very anxious, fears returning to work, and is thinking about seeking disability pay. He has refused to talk to his wife about the incident, because it "makes it worse." Being around large objects also frightens him, since he is worried he will be hit in the head again and he notes that he thinks about the accident constantly. What is the most likely diagnosis? A. Acute stress disorder B. Adjustment disorder C. Factitious disorder D. Generalized anxiety disorder E. Posttraumatic stress disorder

A. The most likely diagnosis is ASD, since this patient's symptoms occurred within 4 weeks of the event and have lasted at least 2 days. If the symptoms had lasted longer than 4 weeks, the diagnosis for a patient with these symptoms would become PTSD. An adjustment disorder is characterized by mood distur- bances (anxiety, depression) in response to a difficult situation, but adjustment disorders do not present with the avoidance symptoms (avoiding talking about the incident) and/or reexperiencing of the traumatic event. Generalized anxi- ety disorder presents with the patient constantly worrying about a wide variety of imagined problems, not connected to an individual traumatic event. Facti- tious disorder is the feigning of mental or physical illness for primary gain.

Parents of a 10-year-old boy note that their son does well with his family until he is not allowed to do something he wants to do. When this occurs, he will get irritable, impulsively aggressive, and agitated for several hours. Once he calms, or gets his way, he is happy and pleasant again. At school he has no trouble focusing but if he does not want to do something he becomes argumentative. Which of the following is the most likely diagnosis? A. Oppositional defiant disorder B. Bipolar disorder-mania C. Attention deficit-hyperactivity disorder combined type D. Major depression E. Generalized anxiety disorder

A. The pattern described best fits oppositional defiant disorder, since the patient can be seen to calm down and be pleasant after he gets his way. A child with bipolar disorder might be seen to have these kinds of outbursts, but the mood dysregulation continues, even after the child "gets his way." Children in the midst of a bipolar disorder never "calm down and be pleasant." Depressed children would also have a persistent mood shift. The symptoms of inattention and distractibility are not present so the patient does not have ADHD. There are no signs of anxiety.

A 28-year-old man is brought to a psychiatrist complaining that he has been hearing voices for the past several weeks. He says that he also heard these voices 3 years ago. He notes that his mood is "depressed" and rates it 3 on a scale of 1 to 10 (with 10 being the best he has ever felt). He does not recall if his mood was depressed the last time he had psychotic symptoms. Which of the following actions should the physician take next? A. Obtain more detailed information about the time course of the psychotic symptoms and the mood symptoms. B. Treat the patient with an antipsychotic agent. C. Treat the patient with an antidepressive medication. D. Request a urine toxicology screening. E. Refer the patient to supportive psychotherapy.

A. The time course of the mood symptoms and psychotic symptoms deter- mines the treatment of the patient because the diagnosis can be schizoaffec- tive disorder versus major depression. Although the patient should undergo a urine toxicology screening, this should not be done until a complete history is obtained so that further targeting of laboratory testing can be accomplished.

An obtunded, young woman is discovered by the police sitting in the middle of the street, and is subsequently brought to the emergency department. She is unable to verbalize any history. Which of the following sets of findings is most indicative of PCP intoxication? A. Nystagmus, muscle rigidity, cannabinoids present on urine toxicology B. Dilated pupils, bradycardia, runny nose C. Pinpoint pupils, tachycardia, orthostatic hypotension D. Ocular nerve palsy, cardiac arrhythmias, pseudobulbar palsy E. Hallucinations, heart block, lower limb weakness

A. The triad of nystagmus, muscle rigidity, and numbness points strongly to PCP intoxication. Other symptoms that can occur include hypertension or tachycardia, ataxia, dysarthria, seizures or coma, and hyperacusis. PCP is very commonly smoked upon application to marijuana. B would be consistent with opioid withdrawal.

A 26-year-old woman comes to see a psychiatrist because she has been taking showers for 6 to 7 hours every day. She explains, "It all starts when I wake up. I am sure I am covered in germs, and if I don't wash, I will get sick. If I don't wash, I get paralyzed with anxiety. Once I'm in the shower, I have to shower in a particular order. If I mess up, I have to start over, and this takes hours and hours. My skin is cracking and bleeding because I spend so much time in the water." Which of the following disorders does this patient most likely have? A. Obsessive-compulsive disorder B. Obsessive-compulsive personality disorder C. Obsessive-compulsive traits D. Paranoid personality disorder E. Schizoid personality disorder

A. This patient demonstrates the classic obsessions, followed by compulsions, of OCD.

A 36-year-old businessman who survived a serious car accident 4 months ago complains of "jitteriness" when driving to work and is currently using public transportation because of his anxiety. He has found himself "spacing out" for several minutes at a time and having difficulty concentrating on his job. He has felt "sad" much of the time, has trouble sleeping at night, has lost 4 lb because of a decreased appetite, and admits that his job performance is slip- ping. Which of the following is the most likely diagnosis? A. Major depressive disorder B. Panic disorder C. Social phobia D. Specific phobia E. Temporal lobe epilepsy

A. This patient exhibits the features of major depression, which commonly occurs with PTSD. In addition, he exhibits two of the three symptom clusters for PTSD, and a clearly traumatic event has preceded his symptoms. It would be unlikely for this patient to suddenly develop an anxiety disorder such as panic disorder, social phobia, or specific phobia. While the "spacing-out" peri- ods are likely to be episodes of dissociation occurring as a result of the trauma, possible neurologic injury should be considered, especially because of his his- tory and the change in his job performance. Also, the patient might be using alcohol to aid in sleeping or to decrease the hypervigilance he has experienced since the accident.

A 60-year-old man is brought to the emergency room by his wife for "confu- sion." She reluctantly confides to the staff that he is a "heavy drinker," that he has drunk up to a case of beer almost every day for the past 30 years. Although he has not changed his alcohol intake significantly, over the past year he has eaten less, preferring alcohol to large meals. She has noticed a gradual weight loss as a result. His last drink was earlier this day. Which of the following would be the most likely finding on the mental status examination of this patient? A. Confabulation B. Delusions C. Elevated affect D. Fluctuating consciousness E. Loose associations

A. This patient has a long history of heavy, regular alcohol use and likely mal- nutrition. A common sequela of this is chronic thiamine deficiency, resulting in Korsakoff syndrome. Korsakoff syndrome is characterized by an antero- grade amnesia; this memory impairment is often (poorly) compensated for by the patient's confabulation, or filling in the missing memories with false information.

A 24-year-old woman gives birth to her first child, a boy. Four days after the delivery, her husband calls his wife's obstetrician to tell him that he is con- cerned about his wife's behavior. The husband states his wife sits by herself in the bedroom almost continually, appearing dazed, but also easily irritated and crying frequently. She tends to the baby only if reminded to do so by her hus- band, and when she holds the baby, she does so stiffly. She is awake most of the night, even when the baby is sleeping. The husband claims she has told him that their baby is "evil" and that she is "satanic" for having the child. Which of the following is her most likely underlying diagnosis? A. Bipolar disorder B. Borderline personality disorder C. Major depressive disorder D. Schizoaffective disorder E. Schizophrenia

A. This woman is most likely suffering from postpartum psychosis. Because of the risk to the mother and infant, postpartum psychosis is a psychiatric emer- gency, requiring immediate hospitalization. Most cases of postpartum psycho- sis are due to bipolar disorder. Major depressive disorder and primary psychotic illnesses such as schizophrenia or schizoaffective disorder are much less likely the cause of postpartum psychosis.

The patient in Question 21.1 is referred to a therapist to begin psychotherapy for his PTSD. Which of the following has been shown to be the most effica- cious treatment for his condition? A. Cognitive-behavioral therapy B. Dialectical-behavioral therapy C. Hypnotherapy D. Insight-oriented therapy E. Psychoanalysis

A. While different types of psychotherapy have been found to be effective in PTSD, various forms of cognitive-behavioral therapy have been the best studied and most useful. Dialectical-behavioral therapy is a specific therapy developed for the treatment of borderline personality disorder. Neither hyp- notherapy nor psychoanalysis has been adequately studied for the treatment of PTSD patients.

Which of the following side effects common to SSRIs is the woman in Question 1.2 most likely to complain of in the future? A. Anorgasmia B. Headaches C. Insomnia D. Nausea E. Tremor

A.Although activation (causing insomnia),gastrointestinal symptoms (including nausea), and tremor are common side effects of SSRIs, only sexual dysfunction generally occurs later in the treatment course (after weeks to months).

A 24-year-old woman states that her alcohol consumption is two glasses of wine or margaritas three times during the week and five vodka drinks on Fri- day and Saturday evenings. This pattern has been going on for nearly a year. She denies any impairment at work as a result of her drinking. She says she is "social" and goes out frequently. Further discussion reveals episodes of blacking out during drinking that have occurred five times in the previous 8 months. She describes these events as "having no recollection of leaving one bar and heading to the next." Which of the following questions is the most specific in screening this patient for alcohol dependence? A. Have you ever attempted to cut down on your drinking? B. How frequently do you drink? C. How much do you drink per day? D. When did you start drinking regularly? E. Do you drink hard liquor or beer and wine only?

A.Althoughquestionsregardingthespecificfrequency,kindofalcohol,amount, and initial use of alcohol are important in establishing a history and pattern of abuse, only asking about attempts to cut back is specific for alcohol dependence, as this question addresses the inability to control alcohol use. The CAGE ques- tionnaire has been validated in screening for alcohol dependence. It uses the mnemonic CAGE. A "yes" answer to two or more of the following questions is a sensitive indicator in diagnosing alcohol dependence. Have you ever felt you should Cut down on your drinking? Have you ever felt Annoyed by someone criticizing your drinking? Have you ever felt Guilty about your drinking? Have you ever had an Eye opener (a drink first thing in the morning)?

A 34-year-old woman presents with a 10-year history of episodes in which she eats large quantities of food, such as eight hamburgers and three quarts of ice cream, at a single sitting. Because of her intense feelings of guilt, she then repeatedly induces vomiting. This cycle repeats itself several times a week. She is extremely ashamed of her behavior but says, "I can't stop doing it." On examination, which of the following physical findings is most likely to be seen? A. Dental caries B. Lanugo C. Muscle wasting D. Obesity E. Body weight at less than the 10th percentile of normal

A.The most likely diagnosis for this woman is bulimia nervosa. Physical findings can include dental caries, a round face caused by enlarged parotid glands, or calluses on the fingers resulting from recurrent self-induced vomiting. Lanugo and muscle wasting result from the severe weight loss characteristic of anorexia nervosa.

A 54-year-old, psychotic, homeless man is brought into the emergency depart- ment by the police for medical clearance, after being picked up for aggressive behavior and disorderly conduct. He has a history of alcoholism and substance abuse, as well as poorly controlled diabetes and hypertension. Which of the fol- lowing symptoms is most specific to a diagnosis of schizophrenia, as opposed to other etiologies of psychosis? A. Auditory hallucinations B. Belief that one has the power of an alien species C. Catatonic symptoms D. Depression E. Inappropriate affect

B. Although all these symptoms can be seen in various psychotic disorders, the presence of a bizarre delusion is the most specific to schizophrenia. Only one psychotic symptom is needed to diagnose schizophrenia if there are bizarre delusions, auditory hallucinations commenting on the patient, or two or more voices speaking to each other.

Patients with which level(s) of mental retardation are capable of holding jobs? A. All levels. B. Mild only. C. Moderate and mild. D. Severe, moderate, and mild. E. Individuals with mental retardation are unable to hold jobs.

B. Although persons with all degrees of mental retardation can require some support to function in the community, those with mild mental retardation are able to hold a job. Individuals with moderate mental retardation are often able to manage small amounts of money and make change. Persons with severe and profound mental retardation have limited abilities to manage themselves and have difficulty learning these skills.

A 17-year-old honors student is brought to the emergency department by his parents. In the last academic quarter, his grades have suddenly dropped, he is irritable with friends and family, he has no energy, he does not go to bed until 1 AM, and he has a poor appetite. He also has auditory hallucinations in which a man's voice tells him that he is a "lazy bastard" and that his family "would be better off with him dead." Which of the following would be the most appropri- ate initial pharmacologic treatment plan? A. Benzodiazepine B. Antidepressant and antipsychotic medications C. Antidepressant medication D. Antipsychotic medication and a benzodiazepine E. Antidepressant medication and lithium

B. An antidepressant combined with an antipsychotic agent is needed for this patient who has major depression with psychotic symptoms. A benzodiaz- epine can address neither the depressive nor the psychotic symptoms. Lithium is an augmenting agent that can be considered later if necessary.

A 24-year-old woman with a diagnosis of bipolar disorder, manic, is treated with a mood stabilizer (lithium) and haloperidol, an antipsychotic. Which of these medications should be discontinued first once her condition has been stabilized? A. Both should be discontinued simultaneously once the patient's condition is stable. B. Haloperidol because of the risk of extra pyramidal side effects. C. Haloperidol because of the risk of habituation. D. Lithium because of the risk of renal damage. E. Lithium because of the risk of weight gain.

B. Because tardive dyskinesia is irreversible, and the risk of its appearance increases with continued use, antipsychotics should be discontinued as soon as the patient's psychotic symptoms remit, and not used again as long as the patient's condition remains stable.

The patient's parents have been searching on the Internet for information about Tourette disorder and its treatment. They have a concern about tardive dyskinesia and would like to have the physician prescribe a medication which will minimize that risk. Which of the following medications, given the par- ents' concern, is the best choice for this patient? A. Pimozide B. Clonidine C. Risperidone (Risperdal) D. Haloperidol E. Clozapine (Clozaril)

B. Clonidine is an alpha-2 agonist that does not cause tardive dyskinesia as a side effect. It is moderately effective in the treatment of vocal and motor tics, although not as effective as some antipsychotics. All of the other agents are antipsychotic medications, and as such, have a risk of tardive dyskinesia associated with their use. Haloperidol, a typical antipsychotic, has the highest risk of tardive dyskinesia of the group, though tardive dyskinesia is a potential complication seen with all antipsychotic medications.

Which of the following complications would be most likely present in the patient in Question 16.1? A. Bradycardia B. Chest pain C. Delirium D. Hypothermia E. Respiratory depression

B. Cocaine intoxication causes numerous physical complications, including chest pain (from coronary vasospasm), tachycardia, diaphoresis, hypertension, and mydriasis. In severe overdose or when combined with other substances, it may cause seizures; however, delirium or respiratory depression are not com- monly seen in intoxication.

An 80-year-old man undergoes an evaluation for dementia versus depression. His caregiver describes a history of gradually worsening depressed mood and confusion, with poor appetite, weight loss, poor self-care, and irritability. On his cognitive examination, he is alert and oriented to person and place but not to time. His concentration is impaired, and he displays poor short-term memory despite adequate recall. His effort is poor overall, and he often responds to ques- tions stating, "I don't know." Which of the following features is more consistent with a depressive illness than with dementia? A. Poor concentration B. Poor effort during the interview C. Poor self-care D. Poor short-term memory E. Poor appetite

B. Difficulty concentrating, decreased self-care, poor appetite, and short-term memory deficits can be seen in both severe depression and dementia in elderly patients. However, during the cognitive examination, patients with depression usually make little effort but have considerable insight into their difficulties, whereas patients with dementia often make considerable effort but display con- fabulation and little insight into their mistakes.

Treatment of the case in Question 55.1 would likely involve administration of which of the following? A. Lorazepam B. Flumazenil C. Chlordiazepoxide D. Disulfiram E. Naltrexone

B. Flumazenil is a benzodiazepine receptor antagonist, which reverses the effects of benzodiazepines. Its use is restricted to use in the emergency depart- ment in instances of benzodiazepine overdose.

A 42-year-old woman is determined to "kick her heroin habit" at home with- out the use of methadone or any other prescription drug. Of the following over-the-counter medications, which is most likely to be of benefit to this patient as she goes through opioid withdrawal? A. Acetaminophen B. Ibuprofen C. Benadryl (diphenhydramine) D. Pseudoephedrine E. Dextromethorphan

B. Ibuprofen can help relieve the muscle cramps that are common to opioid withdrawal.

Bulimia differs from anorexia nervosa in which of the following ways? A. Patients with bulimia tend to be low achievers in academics compared to patients with anorexia. B. Patients with bulimia may not have any symptoms until early adulthood while anorexia typically begins in early adolescence. C. Patients with bulimia are less likely to abuse alcohol and have less emo- tional lability than patients with anorexia. D. Bulimic patients tend to be overweight, whereas anorexic patients are underweight. E. Patients with bulimia are more resistant to receiving help and often must be forced to see a therapist.

B. Patient with both disorders tend to be high achievers but patients with buli- mia tend to be less resistant to getting help, have more alcohol abuse, and have more emotional lability than patients with anorexia, who tend to be more emotionally constricted. Bulimia often has a later onset than anorexia.

A 32-year-old woman is admitted for second- and third-degree burns of her right hand which she attributes to accidentally spilling hot oil while she was cooking dinner. Upon evaluation, the surgeon recognizes the patient as a woman he had treated for a similar burn on the same hand 3 months ago. Further review of her medical records reveal that this is her sixth burn-related injury in 2 years. Which of the following is the most likely diagnosis? A. Conversion disorder B. Factitious disorder C. Malingering D. Hypochondriasis E. Body dysmorphic disorder

B. Patients with factitious disorders consciously fake symptoms to maintain the "sick role." In factitious disorder, there is not obvious secondary gain such as money or avoidance of work as seen in malingering.

A 19-year-old girl is brought to the emergency department by her friends, who are worried that she is not behaving normally. They suspect that she was experimenting with some type of drug, but are unsure what. Which of the fol- lowing syndromes would be most consistent with amphetamine intoxication? A. Flushed face, slurred speech, unsteady gait B. Anorexia, diaphoresis, pupillary dilation C. Prominent hallucinations, pupillary dilation, incoordination D. Miosis, slurred speech, drowsiness E. Hyperphagia, conjunctival injection, tachycardia

B. Symptoms of amphetamine intoxication include anorexia, tachycardia, hypertension, pupillary dilation, and diaphoresis. A = alcohol intoxication, C = hallucinogen intoxication, D = opioid intoxication, E = cannabis intoxication.

Psychotic depression is diagnosed in a 14-year-old boy, and he is treated with an antipsychotic agent, risperidone, and an antidepressant. Three months later, his mood symptoms have resolved, and he is no longer psychotic. Which of the following best describes the next step? A. Both medications should be discontinued via a taper. B. The antipsychotic medication should be discontinued via a taper. C. The antidepressant medication should be discontinued via a taper. D. Both agents should be continued for 6 to 9 months. E. The antipsychotic medication should be stopped immediately.

B. The antipsychotic agent should be discontinued because the psychotic symptoms have abated. The antidepressant should be continued for approxi- mately 6 to 9 months in a patient with the first onset of major depression, and for longer (perhaps indefinitely) for a patient with recurrent depression.

A 29-year-old man is sent to a counselor at his employee assistance program, upon the urging of his supervisor. The patient had been working the night shift, but was recently promoted to a position on the day shift, with new supervisory responsibilities. Subsequently, his job performance has dropped off significantly. The patient states that since his transfer, he has been so nervous at work that he has not been able to think straight. He reports that his mood at home has been good, but that he knows he will fail at the new job because "I have always been such a dope when it comes to working with other people." After several sessions, the counselor diagnoses the patient with avoidant personality disorder. Which of the following would be the most helpful in assisting the patient to manage his anxiety in regard to his new job? A. Tell the patient that he needs to be more confident in his skills during this transition and "suck it up." B. Engage the patient in cognitive therapy to help him deal with his dis- torted thinking. C. Give the patient a beta-blocker to help him control his anxiety. D. Prescribe a benzodiazepine. E. Tell the patient that he is probably not ready for this job if he is this anxious.

B. The goal of cognitive-behavioral psychotherapy in such cases is to help patients critically examine if their assumptions about themselves and other people are correct.

A 48-year-old woman presents to a psychotherapist. The patient lives a very secluded life, largely consumed by working nights as a security guard and taking care of her elderly mother. She complains of feeling lonely, and is aware that she has a great deal of difficulty relating to other people. Which of the following conditions would most distinguish her issues from a person with schizoid personality disorder? A. Family history of a cousin with schizophrenia. B. A desire to engage in interpersonal relationships. C. Lack of hallucinations or delusional thinking. D. Her gender. E. A history of abstinence from alcohol.

B. The hallmark of schizoid personality disorder is a detachment and disinterest in social relationships. This patient is clearly distressed by her lack of social relationships.

A 36-year-old man is referred to a primary care physician for evaluation of his complaints. He is convinced that he has colon cancer despite being told that it is unlikely because of his young age. He occasionally notices blood on the toilet paper and has abdominal cramps when he eats too much. A review of the records demonstrates numerous prior appointments in connection with the same or similar complaints, including repeatedly negative results from tests for occult fecal blood and normal results from colonoscopies. He continues to be worried about dying of cancer and requests another colonoscopy. Which of the following is the most likely diagnosis? A. Body dysmorphic disorder B. Hypochondriasis C. Pain disorder D. Somatization disorder E. Disorder

B. The most likely diagnosis for this man is hypochondriasis. His chief com- plaint is a concern that he has colon cancer. He remains focused on this illness despite prior evaluations with negative results and reassurance from his physi- cian. Although he has several gastrointestinal symptoms (blood in stools and abdominal cramps), he is probably misinterpreting them. His worry is caused by fears of having colon cancer, not about a distorted body image, pain sensa- tions, or numerous physical symptoms.

A52-year-old male executive presents with an onset of depression, early-morning awakening, decreased energy, distractibility, anhedonia, poor appetite, and weight loss for the past 3 months. His symptoms began shortly after he suffered a myocardial infarction. Although he did not experience significant sequelae, he has felt less motivated and fulfilled in his life and work, believing that he is now "vulnerable." As a result, he does not push himself as he used to, and his work output is beginning to decline. A. Bipolar disorder, manic B. Major depression C. Mood disorder due to a general medical condition D. Substance-induced mood disorder E. Adjustment disorder with depressed mood

B. The most likely diagnosis for this man is major depression. He has symptoms typical of the disorder, both depressed mood and neurovegetative symptoms lasting for more than 2 weeks. Although his condition was preceded by a heart attack, it was not a physiologic cause of his depression. Rather, his medical illness (and subsequent feeling of vulnerability) was the stressor that brought on his episode of depression.

A 27-year-old woman states that for approximately 6 months she has believed that Justin Bieber is in love with her. She insists that he has professed his inten- tions to marry her through messages in his song lyrics. She has written numer- ous letters to him and loitered around his home, resulting in several arrests. She is irritated because, although he won't meet with her in person, he often calls her name outside her window when no one else is around. For the past several weeks, she has slept approximately only 2 hours a night but still has enough energy to continuously redecorate her apartment in preparation for her wedding to Mr Bieber. She admits to feeling "on top of the world" because Justin Bieber has chosen her and that she "can't stop talking about it." A. Major depression with psychotic features B. Schizoaffective disorder C. Schizophrenia D. Psychosis secondary to a general medical condition E. Substance-induced psychotic disorder

B. The most likely diagnosis for this woman is schizoaffective disorder. She describes a 6-month history of ideas of reference, delusions, and auditory hallucinations. In addition, she has had clear manic symptoms for the past month, including an elevated mood, a decreased need for sleep, increased energy, increased goal-directed activities, and talkativeness. Although she has symptoms consistent with schizophrenia, she has also had a significant episode of mood disorder during her psychotic illness. Her psychotic symptoms, which preceded and occurred in the absence of mood symptoms, make primary mood disorder (mania) with psychotic features less likely.

A 26-year-old woman presents to her physician with the chief complaint of, "I have epilepsy." She states that for the past 3 weeks she has had seizures almost daily. She describes the episodes as falling on the ground, followed by shaking her arms and legs uncontrollably. These events last for approximately 10 minutes. She is unable to otherwise move during the time, although she denies any loss of consciousness, bladder, or bowel functions. She has never injured herself during these, but as a result she has been unable to continue her job. She is somewhat bothered as she received a promotion 1 month ago. Which of the following is the most likely diagnosis? A. Body dysmorphic disorder B. Conversion disorder C. Hypochondriasis D. Seizure disorder E. Somatization disorder

B. The most likely diagnosis in this woman is conversion disorder ("pseudo seizures"). Conversion disorder patients present with neurological symptoms (eg, sensory deficit, motor weakness, seizures) that are felt to be unconsciously produced and believed to be caused by a psychological conflict or stressor. It is unlikely a seizure disorder given her retention of consciousness and lack of incontinence or injury. Her focus is not on an imagined defect in appearance, on the fear of having a serious illness caused by misperceived body sensations, or on multiple physical complaints.

The patient in Question 41.3 has now been seeing a therapist twice weekly for the last year. The therapist and the patient have a good working alliance. Dur- ing one therapy session, the therapist comes to the session 4 minutes late. He apologizes to the patient, stating that he had an emergency involving another patient. During the session, the patient notes that the therapist "isn't as sharp as some of the therapists I hear on the talk shows." Which of the following defense mechanisms is the patient using? A. Denial B. Devaluation C. Isolation of affect D. Rationalization E. Splitting

B. The patient defends against his feelings of hurt and anger toward the thera- pist by using devaluation. Devaluation along with idealization and denial are considered primitive (lower-functioning) defense mechanisms used by patients with personality disorders such as narcissistic and borderline.

A 32-year-old single male patient is admitted with the provisional diagnosis of psychotic disorder, not otherwise specified, rule-out bipolar disorder. After 10 days, he is finally stabilized on valproic acid 2000 mg daily and aripiprazole 30 mg daily. The nurses are concerned his medications need to be increased or switched as he has been recently sleeping less and more agitated, often pacing the hallways. Upon examination, he admits to feeling "edgy," but he denies racing thoughts, increased energy, paranoia, or delusions. He states, "I just can't stop walking; I feel like I'm going crazy." A. Acute dystonic reaction B. Akathisia C. Neuroleptic malignant syndrome D. Parkinsonism E. Tardive dyskinesia

B. The patient is a young male with psychotic symptoms, rule-out bipolar dis- order, stabilized on valproic acid and aripiprazole, but who recently has had worsening insomnia, anxiety, and restlessness (pacing). Given the improve- ment in his psychiatric symptoms, his current complaints are likely due to akathisia, a feeling of restlessness or anxiety, usually arising several weeks after treatment with antipsychotic medications. Consideration should be given to either decreasing his antipsychotic dose or adding another medication such as a beta-blocker or benzodiazepine.

An 18-year-old boy has benefited from training in social and occupational skills but has been unable to progress beyond the second-grade level in aca- demic subjects. He needs supervision and guidance when under mild social or economic stress. Which level of mental retardation is being described? A. Mild B. Moderate C. Severe D. Profound E. Borderline level of intellectual functioning

B. This description refers to a person with moderate mental retardation. Refer to Table 56-2 for descriptions of the various degrees and functionality of mental retardation.

A 17-year-old high school senior is referred to a psychiatrist by his counselor because of academic difficulty. Although he had always been an honors stu- dent, this past year his grades have quickly dropped, especially in mathemat- ics. When questioned, he reveals the new onset of "superstitions" involving numbers. When presented with certain numbers, he feels compelled to count forward and then backward to and from that number. He becomes anxious about not completing this task, although he is unable to state a particular con- sequence. If interrupted, he must begin all over again. He realizes that there is "no good reason" for his behavior, but is unable to stop it. As a result of this, he not only feels "tortured," but he may need to repeat this year in school. He denies any past psychiatric history. He had an appendectomy at age 15 and takes no medications. He does not drink alcohol or use tobacco products or illicit drugs. Which of the following treatments has demonstrated effectiveness in treating this condition? A. Cognitive-behavioral therapy B. Individual and family cognitive-behavioral therapy plus pharmacotherapy C. Pharmacotherapy alone D. Family therapy E. Psychodynamic psychotherapy plus pharmacotherapy

B. This individual suffers from OCD with his obsessions, compulsions, signifi- cant anxiety, and interference in his academic functioning. Whereas behav- ioral therapy and medications such as clomipramine and SSRIs are helpful in treating OCD, there is evidence that a combination of the two provides the greatest efficacy. There is an absence of studies documenting improvement in OCD solely with psychodynamic psychotherapy.

A 38-year-old man without past medical history presents to his family physi- cian with the chief complaint of "I'm having ulcers." His history, however, does not appear to be consistent with gastritis, ulcers, or reflux. After further questioning, he describes ongoing headaches for 8 months, along with diffi- culty sleeping. Although he denies any specific or recent stressors, he admits to "always being a worry-wart," worrying about many different aspects of his life. When asked to describe his mood on most days, he replies, "I've been depressed my whole life." He only has been taking ranitidine over the counter as needed. He drinks one to two glasses of wine 1 to 2 days per week and denies drug use. Based on his likely diagnosis, which of the following additional psychiatric disorders are most likely to also be present in this patient? A. Antisocial personality disorder B. Dysthymic disorder C. Factitious disorder D. Schizophrenia E. Somatization disorder

B. This patient appears to suffer from generalized anxiety disorder. There is a high comorbidity in this illness, especially with other anxiety disorders, such as panic disorder and phobias. Dysthymic disorder and major depressive disorder are also very commonly associated conditions. Substance use disorders are also not uncommon in patients with GAD.

What is the best treatment option for the patient in Question 54.1? A. Antianxiety medication. B. Antipsychotic medication. C. Hospitalization. D. Psychotherapy. E. Reassure the patient that he is safe.

B. This patient would benefit from small dosage, short-term antipsychotic therapy to manage his delusional thinking. With a paranoid delusion, which is, by definition, a psychotic symptom, antipsychotic drugs are the therapy of choice. Since the patient is not exhibiting dangerous behavior to himself or others, hospitalization is not required. Reassurance, by definition of a delusion (fixed, false belief), will be unhelpful. Antianxiety medication is unhelpful and ineffective with a psychotic disorder, as is psychotherapy.

A 40-year-old woman presents with complaints of not being able to leave her house. For the past 5 years, she has had increasing difficulty traveling far from home. She constantly worries that she will not be able to get help if she "freaks out." In fact, she has had numerous unprovoked episodes of intense fear, associ- ated with shortness of breath, chest pain, diaphoresis, and dizziness, lasting for 20 minutes. She is convinced that if she drives too far from home, she will have an attack and not be able to obtain help. Which of the following is the most likely diagnosis? A. Generalized anxiety disorder B. Panic disorder with agoraphobia C. Panic disorder without agoraphobia D. Social phobia E. Specific phobia

B. This woman most likely has panic disorder with agoraphobia. She experi- ences recurrent spontaneous panic attacks and between attacks worries about having further attacks. She avoids driving away from her home for fear of being unable to obtain help in the event of an attack.

AsthedementiainthepatientinQuestion23.1progressesfurther,hebecomes more agitated and aggressive. Which of the following treatment options would be the most appropriate for his behavior? A. Acetylcholinesterase inhibitor B. High-potency antipsychotic C. Long-acting benzodiazepine D. Low-dose antidepressant E. Low-potency antipsychotic

B. Treatment with a low dose of a high-potency antipsychotic is an appropriate pharmacologic approach to decreasing agitation and aggression in individuals with dementia. Lower-potency antipsychotics should be avoided given their significant anticholinergic and orthostatic side effects. Short-acting benzodi- azepines can also be used but may disinhibit patients and worsen their behav- ior. Long-acting benzodiazepines should be avoided because they can build up in elderly patients and cause oversedation or ataxia. Antidepressants can be useful in treating the depressive and neurovegetative symptoms that are common in dementia, but they would not address the agitation or hostility. Acetylcholinesterase inhibitors can be helpful in improving cognition in the early and mid-stages of Alzheimer disease.

The patient in Question 7.2 returns after 2 weeks. Despite compliance with nortriptyline, her depressive symptoms have since worsened, with the new onset of suicidal ideation without a plan. Upon further questioning, she admits to feeling that unknown individuals are following her and to hearing auditory hal- lucinations calling her name and making derogatory comments. Consideration is given to beginning a trial of ECT. Which of the following is the best indica- tion for using ECT in this patient? A. Multiple medical problems B. Psychotic symptoms C. Recurrent episodes D. Suicidal ideation without a plan E. Significant neurovegetative symptoms

B.Themostappropriatetreatmentforapatientwithmajordepressivedisorder with psychotic features (whether elderly or not) remains ECT or an antidepres- sant/antipsychotic combination. Other indications for ECT are when a rapid response is necessary, such as in an imminently suicidal patient, a catatonic patient, or a patient who is not ingesting adequate food or fluid. Neither recur- rent episodes nor the presence of neurovegetative symptoms are an indication for ECT.

A 52-year-old man is hospitalized for triple CABG and subsequently devel- ops a delirium. He has a history of one closed head injury with loss of con- sciousness, as well as past alcohol abuse with 7 years of sobriety under his belt. Which of the following factors is most likely the prime contributory factor in the development of his delirium? A. His age B. Hospitalization C. Status post-cardiac surgery D. History of head injury E. History of alcohol abuse

C. Advanced age is a major risk factor, with 60% of nursing home residents over age 75 experiencing repeated episodes of delirium. Of all medically ill, hospitalized patients, 10% to 30% exhibit delirium. However, some studies indicate that 90% of postcardiotomy patients experience delirium. Preexisting brain damage, prior history of delirium, alcohol dependence, diabetes, cancer, sensory impairment, and malnutrition may also contribute.

A 28-year-old man presents with a 12-year history of regular alcohol use. Although he has been able to maintain employment as a truck driver, he often drives when "buzzed" in order to make his deadlines. He has been reprimanded on numerous occasions for failure to perform his job adequately, and this has led to increasing conflict with his wife. He denies any recent increase in drink- ing or any withdrawal symptoms, but he does admit to not getting "as drunk as I used to" following consumption of the same amount of alcohol. Which of the following factors in his history is the most specific for alcohol dependence? A. Driving while intoxicated B. Marital conflicts C. Not becoming intoxicated as easily D. Occupational problems E. 12-year history of regular alcohol use

C. Although DWI, marital conflicts, and occupational problems are all criteria for alcohol abuse, only not becoming intoxicated as easily as in the past is a criterion for alcohol dependence. Tolerance for alcohol (experiencing either the same effect with an increased amount or a decreased effect with the same amount), withdrawal from alcohol, and an inability to control alcohol use are the characteristics of alcohol dependence.

A 17-year-old adolescent girl is seen in your office after her friends noticed some strange behavior. The patient reports to you that in addition to some long-term depression issues, she has begun to experience some other disturbing events. She reports that over the last 2 months she has been hearing voices— both at work and at home—of people who she does not think are there. She doesn't recognize these voices. Sometimes they just give a dialogue of what she is doing, but more disturbing to her is when they start saying horrible things about her and tell her to do things she does not want to do. You start her on olanzapine and when she returns in 1 week the voices have been gone entirely for 2 or 3 days. However, she continues to experience severe mood symptoms. Her Hamilton Depression rating scale score places her in the moderate to severe range for depression. What should you do next? A. Inform the patient that these symptoms are the negative symptoms common to the disorder. B. Refer the patient for supportive psychotherapy. C. Treat the patient with fluoxetine (an SSRI). D. Increase the dose of the antipsychotic. E. Add a mood stabilizer to the regimen.

C. Although data are not clear as to the efficacy of administering antidepres- sants to patients with schizoaffective disorder (and depressive symptoms), the continued presence of depressive symptoms makes this treatment worth trying.

Which of the following is the most common cause of mental retardation? A. Fragile X syndrome B. Genetic deficits C. Idiopathic or unknown D. In utero exposure to toxins E. Lead intoxication

C. Although each of the conditions listed is associated with mental retarda- tion, the largest percentage continues to be attributed to idiopathic causes.

A 29-year-old married woman with a prior history of major depression, with postpartum onset, in remission, has given birth to her second child 5 days ago. She describes intense but transient periods of sadness with crying spells since the delivery. While she has been able to enjoy moments with her infant daugh- ter, she worries that she will be "a bad mother." Her sleep is disrupted with resultant fatigue, but her appetite and concentration are adequate. Although she is concerned that her depression will result in harm to her child, she denies any homicidal ideation. She also denies suicidal ideation, paranoia, delusions, or hallucinations. She has no medical problems and is not taking any medi- cations except prenatal vitamins. Which of the following is her most likely diagnosis? A. Bipolar disorder B. Major depressive disorder, recurrent, with postpartum onset C. Postpartum blues D. Postpartum psychosis E. Cyclothymia

C. Although the patient has a history of postpartum depression (a significant risk factor for future episodes) and is suffering from crying spells, fatigue, and guilt, her likely diagnosis is postpartum blues. Postpartum blues usually peaks within the first week after delivery and resolves by 2 weeks. There is no evi- dence of a past or current manic episode (consistent with bipolar disorder) or psychotic symptomatology (consistent with postpartum psychosis). While there should be a high suspicion for developing a recurrence of her major depression, this patient does not have many of the associated neurovegetative symptoms, such as anhedonia, appetite change, difficulty concentrating, or suicidal ideation. In addition, major depressive episodes with postpartum onset typically occur 1 to 6 months postpartum.

Which of the following is the most useful approach for patients with factitious disorder? A. Confronting their feigning of symptoms B. Discharging them from the hospital C. Establishing a therapeutic alliance D. Pharmacotherapy E. Referring them to legal authorities

C. Although there is no specific treatment for factitious disorder, the best way to help these patients is to attempt to establish a therapeutic alliance and a working relationship. Although this can be difficult, only then can the patient's compulsion to feign illness be addressed and dealt with in a psychotherapeu- tic environment. Confrontation is necessary in some circumstances, but if an accusatory or a judgmental manner is employed, patients flee care and begin the cycle again at another hospital. The premature discharge of such patients from the hospital or referral to legal services has the same result, although in cases of factitious disorder by proxy (where a caretaker simulates illness in a child), referral to child protective services is necessary because this behavior is considered a form of child abuse. Pharmacotherapy use should be limited un- less the patient has a comorbid axis I disorder. Due to abuse potential, medica- tions should be used with caution.

A 23-year-old medical student makes lists of all the tasks that he must accom- plish each day. He spends hours studying and refuses to go out with his col- leagues even when there are no tests on the immediate horizon, preferring to spend his time looking at specimens in the laboratory. He keeps meticulous notes during all his classes and prefers to attend every lecture, not trusting his colleagues to take notes for him. He is doing well in school and has a girlfriend who is also a medical student. Which of the following disorders does this stu- dent most likely have? A. Obsessive-compulsive disorder (OCD) B. Obsessive-compulsive personality disorder C. Obsessive-compulsive traits D. Schizoid personality disorder E. Paranoid personality disorder

C. Although this student clearly demonstrates some traits of obsessive- compulsive behavior, his social and occupational functioning both are good, which rules out the personality disorder.

A 35-year-old woman with somatization disorder comes to see a new doctor. Prior to seeing this physician, she obtained complete evaluations from at least four clinics. The most important part of the treatment plan for this patient is which of the following? A. A trial of analgesics. B. Antidepressant medication. C. Establishing a schedule for regular visits. D. Explain symptoms are due to psychological factors. E. Intensive psychodynamic psychotherapy.

C. Analgesics are generally not helpful in treating these patients, and they probably cannot be relied on to take psychotropic medications. Patients with somatization disorder have difficulty recognizing affects and so are not can- didates for dynamic therapy, especially early in their treatment. The most important treatment intervention is establishing a relationship with one clini- cian and scheduling brief, regular visits. Increasing the patient's awareness of possible psychological factors involved in the symptoms is a long-range goal and should be done when and if the patient is willing to see a mental health professional.

A 22-year-old, single graduate student with narcissistic personality disorder is admitted to a hospital after a car accident in which his right femur is fractured. A medical student has been assigned to follow the patient, but when she enters the room and introduces herself as a medical student, the patient states, "Oh, I wouldn't let a medical student touch me—I need someone with much more experience than you." Which of the following statements by the medical stu- dent is most likely to lead to a successful interview with this patient? A. I know this will be boring for you, but it's just one of the things that you will have to put up with in the hospital. B. I know you must be scared to be in the hospital, but you will be safe here. C. I'm told that you are a very articulate person, and so I'm hoping you'll teach me what I need to know. D. I understand that you think you deserve only the best, but I have been assigned to you. E. Please don't make this difficult, I have to interview you as part of my job.

C. Appealing to the patient's narcissism by being admiring most often de- escalates the patient as well as improves the therapeutic alliance in these cases.

A 6-year-old boy with an early, ongoing history of distractibility, hyperactivity, and impulsivity is diagnosed with ADHD. He is treated with methylphenidate. Three weeks later he is brought in and his inattention and hyperactivity are much better. The mother also notes that he has a small bald spot from where he has begun repeatedly rubbing his head. You periodically observe him to suddenly raise his hand to the spot, rub back and forth once, and put his hand down. There is no rash but the area is hairless. The most likely diagnosis is which of the following? A. Alopecia secondary to stimulant B. Drug allergy with contact dermatitis C. Stimulant-induced complex motor tic D. Scabies E. Attention-seeking behavior

C. Atomoxetine appears to have a lower potential for abuse than Ritalin. Initial use can produce a feeling of sleepiness and grogginess, and the drug does not appear to exhibit its effects for up to 3 weeks. It is not recommended to take atomoxetine on an empty stomach, as side effects of the drug include nau- sea and vomiting. Atomoxetine is not currently available in a generic form. One of the advantages of this drug is that it affords 24-hour control of ADHD symptoms (ie, it has a longer half-life than Ritalin).

A 16-year-old girl has been admitted with a 3-week history of sudden irritabil- ity, impulsive buying, and disappearing at night with older men. Her need for sleep is decreased; she has flight of ideas and grandiose thoughts about being an advisor to a presidential candidate. Routine admission labs indicate she is pregnant. Which of the following is a statement that should be made to her parents? A. Treatment with an SSRI antidepressant is a reasonable alternative to mood stabilizers in a pregnant girl. B. Given the fact that she is pregnant, she should be kept secluded on an inpatient unit during the first trimester of pregnancy with no medications. C. An atypical antipsychotic may be the best choice for managing both psychotic features and mood disturbance associated with her bipolar disorder, especially during the first trimester. D. Psychotherapy will have little role in the treatment of her bipolar disorder. E. Lithium, divalproex, and carbamazepine are all reasonable first-choice mood stabilizers for this patient.

C. Atypical antipsychotics are good choices for controlling mania in pregnancy given the strong teratogenic effects of most mood stabilizers. SSRIs will only make the mania worse. The patient will need psychotherapy focused on helping her sort out her feelings about the pregnancy.

Which type of personality disorder is most likely to occur comorbidly with factitious disorder? A. Antisocial B. Avoidant C. Borderline D. Obsessive-compulsive E. Schizoid

C. Borderline personality disorder is not uncommon in patients with facti- tious disorder. Individuals with either of these disorders often have similar his- tories of abuse, molestation, and emotional neglect. Patients with borderline personality disorder also act out their internal psychological conflicts on an interpersonal level, and they display the chaotic, labile affective state seen in factitious disorder.

The patient described in Question 46.2 can best be distinguished from Rett disorder because of which of the following? A. Social difficulties B. Lack of language development C. Absence of a period of normal development D. Evidence of mental retardation E. Sex of the child

C. Both syndromes show evidence of presence prior to the age of 36 months; however, in Rett disorder, the child has an early phase of normal development then gradually loses those skills toward further decline.

A 37-year-old woman is referred to your office for psychotherapy. She is pre- occupied with thoughts of her husband leaving her and being left alone with nobody to take care of her. She feels sad and hopeless when alone, lacks self- confidence, and has great difficulty making decisions. You believe she suffers from a personality disorder. In which cluster does the patient's most likely diag- nosis belong? A. Cluster A B. Cluster B C. Cluster C D. Cluster D E. Cluster E

C. Cluster C can also be remembered as the "sad" cluster (see Case 6). It includes dependent, avoidant, and obsessive-compulsive personality disorders. The clusters can be remembered by the words "mad" (cluster A—characterized by odd or eccentric behavior), "bad" (cluster B—characterized by dramatic or emotional behavior), and "sad" (cluster C—characterized by anxious or fearful behavior).

A 39-year-old man with antisocial personality disorder, incarcerated for life after murdering a man, has a multitude of somatic complaints over the course of several years. Yearly physical examinations never show anything physically wrong with him, yet he complains of a variety of aches and pains, neurologic symptoms, and gastrointestinal distress. He does not enjoy the time he spends in the jail's infirmary. Which of the following is the most likely explanation for this patient's complaints? A. He is malingering. B. He has developed a psychotic disorder. C. He has developed a somatization disorder. D. He has an undiagnosed physical illness. E. He has an undiagnosed anxiety disorder.

C. Development of a somatization disorder becomes more common in patients with antisocial personality disorder as they grow older. There is no evidence of secondary gain here (which rules out malingering), nor is there evidence of psychotic thinking. Physical examinations have all been negative (making a physical illness less likely) and the complaints are all around somatic symp- toms, making a pure anxiety disorder less likely as well.

A 25-year-old man with schizotypal personality disorder comes to his psychia- trist with a chief complaint of a depressed mood. He notes that since losing his job as an astrologer, he has been depressed and unable to sleep. He states that although his mood is usually fairly low (4 out of a possible 10), it has lately been a constant 2. The patient also notes problems with concentration and energy level and has experienced several crying spells. He reports he had pre- monitions that certain foods could heal him, so he has been mixing "magical potions" and eating "magical foods." A mental status examination reveals an oddly dressed man with constricted affect, ideas of reference, unusual beliefs, and some mild paranoia. Which of the following medications is most likely to be helpful to this patient? A. Zolpidem (Ambien) for insomnia B. Divalproex sodium for mood disturbance C. Escitalopram for depressive symptoms D. Risperidone for paranoia E. Ziprasidone for ideas of reference

C. Escitalopram is a selective serotonin reuptake inhibitor (SSRI) useful in the treatment of depression. Patients with schizotypal personality disorder who have either a depressive component to their illness or a secondary super- imposed major depression (as is the case with this patient) should be treated with antidepressants. Ziprasidone (Geodon) and risperidone (Risperdal) are atypical antipsychotics that would be effective if the patient was having a transient psychotic episode, but he is not. Divalproex sodium is an antiepi- leptic agent used as a mood stabilizer to treat conditions such as mania.

The acute crisis of the woman in Question 55.1 is averted. She is subsequently hospitalized for detoxification. Which of the following benzodiazepines is most likely to cause a withdrawal syndrome? A. Chlordiazepoxide B. Chlorazepate C. Alprazolam D. Diazepam E. Lorazepam

C. Generally speaking, shorter-acting agents are more likely to cause a with- drawal syndrome. Alprazolam is the shortest-acting benzodiazepine listed.

Which of the following treatments is best employed to treat the comorbid depressive symptoms of an adolescent with CD? A. Multisystemic therapy B. Attendance in group therapy C. An antidepressant medication D. Treatment of the family to address the underlying reasons for the depression E. Helping the adolescent change schools

C. If the criteria for a comorbid condition are met, this disorder should be the first target of psychopharmacologic intervention.

A 62-year-old man presents to his primary care physician at the insistence of his wife. The man states that there is nothing wrong with him, but that he has not been sleeping nearly as much as he used to, often needing less than 2 hours of sleep a night. His wife notes that he has been exceedingly irritable, has been charging excessive amounts of money on their credit cards, and has been talking about running in a marathon, although he has never expressed such an interest before. The patient has no previous psychiatric or medical history. He denies the use of drugs or alcohol. Which of the following courses of action should the physician take first? A. Admit to the hospital. B. Order a urinalysis for drugs of abuse. C. Perform a complete physical examination. D. Start a mood-stabilizing drug. E. Start an antipsychotic drug.

C. In a patient of this age, with no previous psychiatric or medical history, general medical conditions responsible for this new-onset behavior should be ruled out before an episode of bipolar mania is considered. While admission to the hospital might well be necessary to control the destructive behavior, making the diagnosis first is the primary concern. Both a mood stabilizer and/ or an antipsychotic may well also be necessary, but again, if the mood problem is secondary to a general medical condition, treating it may resolve the psychi- atric symptoms by itself.

A 17-year-old boy presents with a complaint of his "legs giving out" for 1 week. During each episode he experiences a generalized painful sensation and within a few seconds he becomes weak and unable to move his arms and legs. The episodes last a few minutes. The patient is in the 11th grade earning Bs and Cs. His parents have recently separated after a long period of verbal abuse toward each other. His physical examination and neurologic workup are unremark- able. Laboratory studies are normal. Which of the following characteristics most distinguishes his diagnosis from other somatoform disorders? A. Symptoms are not fully explained by a medical cause. B. Symptoms are not intentionally produced. C. Symptoms involve only motor or sensory neurologic deficits. D. Psychological factors are related to symptom production. E. Secondary gain is involved.

C. In all somatoform disorders, the symptoms, pain, deficits, or preoccupation with a serious illness are not fully explained by a medical condition or are out of proportion. None of these symptoms are intentionally produced, as in facti- tious disorder or malingering. Psychological factors (conflicts or stress) are asso- ciated with the creation, exacerbation, and/or maintenance of the symptoms in all these disorders. Although somatization disorder involves a pseudoneu- rologic symptom, multiple, additional symptoms are also required. Conversion disorder is distinguished by having only a motor or sensory deficit.

A 39-year-old man presents to the emergency room at the behest of his girl- friend, who reports that he has not slept in 3 days. The patient speaks extremely rapidly, and switches topic so frequently as to be incomprehensible. His affect is happy and elevated, but he quickly snaps and becomes belligerent when he is accidentally bumped by a nurse. Which of the following symptoms would most likely distinguish this patient's presentation from PCP intoxication? A. Disorganized thoughts B. Hostile or violent behavior C. Nystagmus D. Hallucinations E. Pressured speech

C. Individuals with mania and PCP intoxication can have hallucinations, dis- play hostility, and have disordered thoughts; nystagmus is commonly associated with PCP use but not with mania.

A 24-year-old woman is called into the head office of the agency where she works and told that her chronic lateness in completing her assignments will result in her dismissal if she does not change her behavior. The patient really loves her job, and the news comes as a major blow. That night at home, she tells her boyfriend in great detail about each and every step of the meeting and spends the entire night thinking about her job. The boyfriend tells her that she does not "look" particularly upset. Which of the following defense mecha- nisms is being used by this woman? A. Undoing B. Displacement C. Intellectualization D. Rationalization E. Splitting

C. Intellectualization is a defense mechanism by which an individual deals with emotional conflict or stressors with an excessive use of abstract thinking to control or minimize disturbing feelings. Because the stressors have been successfully defended against in this instance, the patient does not appear par- ticularly distressed.

A 28-year-old woman comes to your clinic after being seen by her primary care doctor and several specialists. She complains that for the last 2 years she has experienced headaches, back and joint pain, abdominal pain with nausea and bloating, numbness and tingling in her upper extremities, and irregular men- ses. No physical cause for her symptoms can be found. The patient is insistent that she just can't work anymore due to "her pain" and wants you to sign a sick leave request form. Which of the following might help rule out somatization disorder? A. Age of this patient. B. Presence of four pain symptoms. C. Presence of secondary gain. D. Symptoms can't be explained by a medical condition. E. Symptoms lasting for 2 years.

C. Intentionally produced symptoms rule out somatization disorder. When external (secondary) gains such as avoiding work are present, malingering should be considered, and if malingering is indeed the diagnosis, it is likely that the symptoms are being produced intentionally. The other signs/symptoms on this list are consonant with a somatization disorder, and thus would not help rule it out.

An elderly woman presents to the emergency department due to a hip frac- ture. She reports that she "hasn't been feeling very well," recently, and is vague and hard to pin down regarding details. You think that there might be the odor of alcohol on her breath, and suspect alcohol abuse versus depen- dence. Which of the following findings would be most supportive of your concern? A. A healed scar from a previous fall several years ago B. Microcytic anemia C. Elevated gamma-glutamyl transpeptidase D. Slightly elevated aspartate aminotransferase (AST), with normal alanine aminotransferase (ALT) E. Mini-Mental State Examination score of 28/30

C. Laboratory tests would commonly show macrocytic anemia, elevated liver transaminase levels—particularly gamma-glutamyl transpeptidase in alcoholics.

A 15-year-old boy is brought to the emergency department by the police due to violent, psychotic behavior. Phencyclidine intoxication is confirmed via urine toxicology. Which of the following treatment interventions is associated with the lowest risk of adverse complications? A. Low-potency, traditional antipsychotic agents to treat hallucinations B. Ammonium chloride to acidify the urine and increase clearance of the drug C. Benzodiazepines for agitation D. Gastric lavage to remove unabsorbed, excess drug E. Full-leather restraints to prevent harm to self or others

C. Low-potency traditional antipsychotics may worsen intoxication syndrome via anticholinergic side effects. Acidification of the urine has been found to be ineffective, and increases the risk of acute tubular necrosis due to myoglobinuria in the rhabdomyolysis. Gastric lavage is contraindicated due to risk of emesis and aspiration, while restraints may lead to muscle breakdown.

A 10-year-old girl is brought in for treatment by her father following the death of her mother 6 weeks previously because of an unexpected heart attack. The father is worried because the child is not sleeping well, has lost 7 lb because of a decreased appetite, seems to be tired much of the time, and is preoccupied with memories of her mother. He notes that she cannot concentrate on her usual favorite television shows and has lost interest in many of her previous social activities. The patient reports that she deeply misses her mother, but she also smiles in recalling many pleasant memories of their life together. Which of the following is the most likely diagnosis? A. Adjustment disorder with depressed mood B. Major depression C. Normal bereavement D. Sleep disorder E. Dysthymic disorder

C. Normal bereavement. This patient's symptoms fulfill the criteria for major depression, but she is in the very early stages of bereavement when such behav- ior is considered normal. Her sleep problems are part of bereavement. If her symptoms do not diminish in the subsequent 4 months, the clinician should make a reassessment for major depression.

Which of the following factors is most likely associated with a more positive prognosis for autistic disorder? A. Physical development/performance IQ B. Family socioeconomic status C. Language development/verbal IQ D. Presence of a seizure disorder E. Sibling order

C. One of the best predictors of the ability to improve the development of autistic children is the extent to which they have, or have begun to develop, language skills. This ability is often measured in verbal IQ testing.

Which statement best characterizes the difference between patients with avoidant personality disorders and those with schizoid personality disorders? A. Patients with avoidant personality disorders have fewer friends than those with schizoid personality disorders. B. Patients with avoidant personality disorders have higher self-esteem than those with schizoid personality disorders. C. Patients with avoidant personality disorders would like to have friends more than patients with schizoid personality disorders. D. Patients with avoidant personality disorders are better at accepting criti- cism than patients with schizoid personality disorders. E. Patients with avoidant personality disorders are less anxious than are patients with schizoid personality disorders.

C. Patients with avoidant personality disorders would desperately like to have social relationships, but they are afraid of criticism and/or rejection.

A patient with schizoid personality disorder comes to his primary care physician with chief complaints of polyuria and polydipsia. He is found to have insulin- dependent diabetes. Which of the following interventions by the physician is likely to be most well received by this patient? A. Asking the patient to bring in a relative so that he can describe the treat- ment regimen to both of them at the same time. B. Referring the patient to a therapist so that he can talk about the difficult nature of the diagnosis. C. Giving the patient detailed written information about the disease and tell- ing him that the physician will be available to answer any questions. D. Referring the patient to a group that helps its members learn about diabetes and to better deal with their illness. E. Scheduling frequent appointments with the patient so that all the treat- ment details can be explained on a one-to-one basis.

C. Patients with schizoid personality disorder generally prefer to keep social interaction to a minimum. They do well with technical information.

A 16-year-old adolescent girl is incarcerated in a juvenile detention facility. She is currently charged with theft, apparently to support her and her boy- friend's drug habit. She has had multiple involvements with child and family services for running away from home, where she apparently had been sexually abused by her mother's boyfriend. She has a diagnosis of posttraumatic stress disorder (PTSD). Prior to the onset of the abuse, she was doing extremely well in school, in an accelerated program. Which of the following factors speaks most strongly against a diagnosis of antisocial personality disorder? A. Her concurrent diagnosis of PTSD B. Her gender C. Her age D. Antisocial acts committed to support a drug habit E. Apparent high intelligence

C. Personality disorders cannot be diagnosed before 18 years of age. Antisocial personality disorder should not be overlooked in females, even though it is much more common in males. Antisocial actions committed solely during psychotic or manic episodes, or to support a drug habit, would not support a diagnosis of antisocial personality disorder.

Treatment of ASD should focus primarily on which of the following? A. Biofeedback B. Debriefing the individual about the even C. Mobilizing social supports D. Pharmacologic treatments, such as selective serotonin reuptake inhibitors E. Psychotherapy

C. Pharmacologic treatments, psychotherapy, and biofeedback are primarily interventions for PTSD; the results of research are currently unclear regard- ing the benefits of debriefing. Mobilizing social supports is the most effective intervention in treating patients with ASD.

The patient in Questions 22.1 and 22.2 returns to your office for a medication check. Sertraline was started 4 months ago. The dose has been increased twice and the patient has been taking 200 mg for 2 months. She does not feel the medication has provided any relief to her symptoms, but she has tolerated the medication well. What would be the next step for this patient? A. Augment by adding another medication. B. Discontinue and begin a tricyclic antidepressant (TCA). C. Discontinue and begin another selective serotonin reuptake inhibitor (SSRI). D. Give the drug more time. E. Increase the dose.

C. SSRIs are generally tolerated better by patients and are often beneficial in treating dysthymia. Two 200 mg of sertraline is the maximum dose and 6 to 10 weeks is considered an adequate trial, so another SSRI should be tried before switching to another class. Augmentation is usually done when the patient has a partial response to an initial medication.

A patient with ADHD is treated with methylphenidate during the school year. After several months of treatment, his teachers and parents note that he has developed both motor and vocal tics. What should be the first course of action for these symptoms? A. Begin treatment with haloperidol. B. Discontinue the use of methylphenidate. C. Switch medication to atomoxetine. D. Reduce the dose of methylphenidate. E. Administer an anticonvulsant.

C. The development of tics as a side effect of stimulant medication is rela- tively common. While these tics diminish in severity or cease when the dose is reduced, reduction of dose often results in an increase of ADHD symptoms. Because atomoxetine is quite effective for treating ADHD without stimulating tics, a trial on atomoxetine should be considered. If ADHD symptoms and/or tics continue on atomoxetine then addition of clonidine or guanfacine should be considered.

Adjustment disorder is diagnosed in a 45-year-old woman who was fired from a job she held for 20 years. She undergoes supportive psychotherapy. Nine months later, she is seen by her physician, but none of her symptoms have resolved. During this time, she has found another job that is similar to her first position in duties and salary. Which of the following is the most likely diagnosis? A. Adjustment disorder B. Posttraumatic stress disorder C. Major depressive disorder D. Bipolar disorder E. Schizoaffective disorder

C. The duration requirement for symptoms occurring after the stressor resolved is met for a major depression. Since the stressor has been removed, an ongo- ing adjustment disorder would be an incorrect answer. There is no evidence for psychosis or manic moods, so options relating to bipolar or schizoaffective disorder would be incorrect. PTSD would not be a viable option because the patient has not had a life-threatening stressor occur.

A 33-year-old man has the chief complaint of "I'm going to have a heart attack like my father." He explains that his father died of a myocardial infarction at 45 years of age. He is convinced that he is experiencing anginal attacks con- sisting of nervousness, sweating, palpitations, flushing, and numbness in his hands lasting for several minutes. He is anxious about having these symptoms and, despite negative results from a cardiology workup, remains certain that he will suffer a heart attack. His behavior and lifestyle have not been otherwise affected. Which of the following is the most likely diagnosis? A. Generalized anxiety disorder B. Panic disorder with agoraphobia C. Panic disorder without agoraphobia D. Social phobia E. Specific phobia

C. The most likely diagnosis for this man is panic disorder without agoraphobia. He displays characteristic features of panic attacks, such as recurrent episodes of anxiety associated with physical symptoms. These episodes are spontaneous, and he worries about the consequences of having an additional attack, namely, a myocardial infarction. However, he does not avoid places, and his behavior is otherwise unaffected.

Which of the following treatment approaches is the most appropriate for the patient in Question 39.1? A. Antidepressant medication B. Antipsychotic medication C. Continued observation D. Electroconvulsive therapy E. Mood stabilizer

C. While this patient may develop a recurrence of major depression requir- ing antidepressant therapy, postpartum blues is self-limiting, not necessitat- ing formal treatment. Continued observation and reassurance are all that are recommended. Postpartum psychosis requires treatment with an antipsychotic in combination with an antidepressant or a mood stabilizer, or a course of electroconvulsive therapy.

A 46-year-old man presents with a long-standing belief that his thoughts are being taken from his head and used to create a blockbuster movie. He is cer- tain that the government is involved because they often communicate with him through a microchip they have implanted in his brain. Although he feels frustrated at being taken advantage of, he denies any significant depressive symptoms and is often able to enjoy playing cards with his peers at the group home. A. Major depression with psychotic features B. Schizoaffective disorder C. Schizophrenia D. Psychosis secondary to a general medical condition E. Substance-induced psychotic disorder

C. The most likely diagnosis for this man is schizophrenia. He has been suffer- ing from psychotic symptoms including delusions and auditory hallucinations for more than 6 months. Although he can have brief periods of depressed mood, he does not have a history of major mood disorder.

An 80-year-old woman without a psychiatric history is examined after a left-sided cerebral vascular accident that has left her paralyzed on her right side. Since her stroke, she complains of an absence of pleasure in anything that she formerly enjoyed. She describes frequent crying spells, increased sleeping, a decreased appetite with weight loss, and feelings of hopelessness and helplessness. A. Bipolar disorder, manic B. Major depression C. Mood disorder due to a general medical condition D. Substance-induced mood disorder E. Adjustment disorder with depressed mood

C. The most likely diagnosis in this case is mood disorder caused by a general medical condition, namely, a cerebral vascular accident. The patient has obvi- ous symptoms of a depressive illness, including anhedonia. These symptoms also have a clear temporal relationship to her stroke, which has left her with significant motor deficits. Cerebral vascular events are well known to result in depression.

A39-year-old woman presents with 1month of a gradually worsening depressed mood, with increased sleeping, low energy, and difficulty concentrating, but no appetite or weight changes. Her medical history is significant for multiple sclero- sis, but she is currently not taking any medication. Her mental status examina- tion is notable for psychomotor slowing and a depressed and blunted affect. Her physical examination demonstrates several different sensory and motor deficits. A. Bipolar disorder, manic B. Major depression C. Mood disorder due to a general medical condition D. Substance-induced mood disorder E. Adjustment disorder with depressed mood

C. The most likely diagnosis in this case is mood disorder due to a general medical condition, namely, multiple sclerosis. Although this woman displays the characteristic symptoms of an episode of major depression (depressed mood, increased sleeping, low energy), she does not exhibit the appetite or weight changes commonly seen in this illness. Steroids can often cause mood symptoms such as depression or mania, but she is currently not taking any medication. The results of her physical examination are also consistent with a flare-up of her multiple sclerosis and demonstrate a temporal relationship to her depression. The central nervous system white matter lesions seen on imaging are known to be associated with a depressive state.

Which of the following features must be present in a patient's history for schizotypal personality disorder to be diagnosed? A. The patient must have a history of active substance use. B. The patient must have a history of auditory hallucinations. C. The patient must have a history of cognitive and perceptual distortions. D. The patient must have a history of intense, short-lived friendships. E. The patient must have a history of suicidal ideation.

C. The odd quality in which these patients perceive and think about the world is one of the diagnostic criteria for schizotypal personality disorder. Answers A, B, and E are symptoms of a variety of axis I disorders, but do not point to any particular personality disorder. Answer D is a characteristic seen in borderline personality disorder patients, although it is not a diagnostic criterion per se.

A 14-year-old girl's family brings her to treatment against her wishes. The fam- ily reports she has a very short temper, argues with them and teachers at school daily, often refuses requests at both settings, and usually blames her teachers or her parents for the behaviors she is showing. She denies being depressed, anxious, or any psychotic symptoms. With this information, what might be the best diagnosis? A. Antisocial personality disorder B. Borderline personality disorder C. Oppositional defiant disorder D. Conduct disorder E. Posttraumatic stress disorder

C. The patient does not yet meet criteria for conduct disorder, which might include additional signs such as aggression to people or animals, destruction of property, theft, or significant rule violation. Both conditions cause impairment in social, family, and academic functioning.

A 4-year-old boy who is an only child starts preschool. His parents are quite nervous about this and describe themselves as protective and over-involved. However, they are looking forward to seeing him more involved with school, as he has had little peer interaction prior to this because he is an only child, and he has never shown much of an interest in interacting with others. He has never used many words which the parents attribute to his isolation as an only child. They tell the teacher he has always done best with a strict schedule and doesn't tolerate changes well. He has always seemed to have a very nar- row repertory of play—focusing primarily on spinning objects such as tops and balls. He comes to the classroom for the first time and runs right to these toys and does not say goodbye to his mother, nor even acknowledge she is leaving. Other children attempt to play with him but his response is to either ignore them or get angry at their advances. This last response in someone with autism might best be described as which of the following? A. Rigidity B. Stereotyped behavior C. Lack of social reciprocity D. Poor language development E. Obsessional thinking

C. The patient's lack of response and then inappropriate response to the social advances of his peers might best be considered a symptom of impairment of social reciprocity. Social reciprocity involves the appropriate sensing and then interpretation of verbal and nonverbal social cues that guide our interactions.

Which of the following is most likely the motivation behind the behavior displayed in factitious disorder? A. The motivation is unconscious and thus the patient is unaware of it. B. Desire to avoid jail. C. Desire to take on the patient role. D. Desire to obtain compensation. E. Desire to obtain narcotics.

C. The primary desire in factitious disorder is to assume the sick role and be taken care of. In contrast, in malingering, the motivation is to achieve a tan- gible gain (such as avoiding work, school, or a prison sentence) or to obtain narcotics or financial compensation. If the motivation is unconscious, conver- sion and other somatoform disorders would need to be considered.

A 26-year-old woman with a history of major depressive disorder and general- ized anxiety disorder presents to the emergency department with her husband complaining of anxiety, shortness of breath, palpitations, and "feeling like I am going to die." Her symptoms began earlier in the day, but over the past 2 hours have gotten significantly worse. She has no significant past medical history, smokes half a pack of cigarettes per day, and her only medications are oral con- traceptives and venlafaxine. On examination, she speaks in short sentences and appears diaphoretic. Her pulse rate is 160 beats per minute, blood pressure 104/64 mm Hg, and respiratory rate is 32 breaths per minute. An ECG shows sinus tachycardia. What is the most appropriate next step? A. Adminster lorazepam. B. Start buspirone. C. Order a high-resolution CT of the chest. D. Tell her she is having a panic attack and would benefit from CBT. E. Inquire about current social stressors.

C. This 26-year-old woman is presenting with classic signs of a pulmonary embolus (tachycardia, hypotension, and tachypnea). Additionally, her use of cigarettes and oral contraceptives places her at greater risk for PE than the general population. Although she has a psychiatric history, dismissing her complaints as a panic attack would be negligent. Lorazepam will not aid in diagnosis and may result in clinical deterioration. Inquiring about social stressors is not appropriate in the setting of hemodynamic instability. A high-resolution CT of the chest is indicated.

A 27-year-old man is admitted for acute psychotic symptoms, consisting of command hallucinations to harm others, paranoid delusions, and agitation. He is begun on olanzapine 30 mg daily. After several days, he becomes calmer but more withdrawn. When approached by the nurses, he is found to be lying in bed, eyes open but not responsive. He is noted to be sweating but is resistant to being moved. His vital signs demonstrate a temperature of 101.4°F, blood pressure 182/98 mm Hg, pulse 104/min beats per minute (bpm), and respira- tion 22 breaths per minute. A. Acute dystonic reaction B. Akathisia C. Neuroleptic malignant syndrome D. Parkinsonism E. Tardive dyskinesia

C. This acutely psychotic patient has been started on an antipsychotic med- ication, namely olanzapine, and he has now developed acute mental status changes, diaphoresis, rigidity, and fluctuating vital signs. These are signs and symptoms consistent with neuroleptic malignant syndrome, a medical emer- gency. Antipsychotic medications should be discontinued immediately, and supportive measures need to be employed.

A20-year-old woman comes to see a psychiatrist at the insistence of her mother, who states that her daughter just "isn't herself." The patient has dressed in brightly colored clothes and worn large amounts of makeup for the past 3 weeks. She acts overtly seductive toward her colleagues at work, is more distractible, and is easily irritated. She also sleeps less, claiming that she "no longer needs it." Which of the following diagnoses best fits this patient's presentation? A. Histrionic personality disorder B. Borderline personality disorder C. Bipolar disorder, mania D. Narcissistic personality disorder E. Delusional disorder

C. This patient has a new onset of behavior that is unlike her usual personality. It includes dressing in loud clothing and wearing lots of makeup, as well as being seductive. She is distractible, irritable, and needs less sleep than usual. All these symptoms point to a manic episode (assuming neither a medical condition nor a substance can account for the sudden change in functioning).

Over the next 2 years, this patient continues to struggle and is arrested for truancy, fighting, and possession of drug paraphernalia. Given this informa- tion, what might we best be able to say about patients with this diagnosis in general? A. Patients are more likely to have parents with a history of schizophrenia. B. Patients are more likely to be female. C. Patients are more likely to have parents who have antisocial personality disorder and alcohol dependence. D. The presence of this pathology is independent of social class. E. The course of this disease is time limited and she is likely to grow out of it.

C. This patient now fits the criteria for conduct disorder. Conduct disorder is more common in children of parents with antisocial personality disorder and alcohol dependence than it is in the general population.

A 36-year-old man comes to a physician's office with a chief complaint that "people are out to hurt me." Despite being reassured by his wife that this is untrue, the patient is convinced that men are observing his behavior and actions at home and at work, using telescopic lenses and taping devices. He has torn apart his office on more than one occasion looking for "bugs." The patient's wife says that this behavior is relatively new, appearing somewhat suddenly after the patient was robbed on the way to his car approximately 6 months previously. Which of the following symptoms best describes what the patient is experiencing? A. Ideas of reference B. Hallucinations C. Paranoid delusions D. Paranoid ideations E. Thought disorder

C. This patient's problem is more than mere suspiciousness; he has full-blown paranoid delusions: fixed, false beliefs. Paranoid ideation is mere suspicious- ness—the worry that harm is meant by others. People with paranoid ideation may often be consoled or reassured by a trusted friend, and they do not often act on these suspicious. By contrast, people with paranoid delusions have fixed (ie, they are not able to be reassured) and false beliefs that others mean them harm. These patients may act on these beliefs as well, for example, this patient tearing apart his office looking for "bugs."

Youareconsultedtoevaluatean84-year-old,widowedwomanonthemedicalward. She has a prior history of major depressive disorder, recurrent, and she was admit- ted for a syncopal episode. She was found to be extremely malnourished, hypoten- sive, and in acute renal failure. She has not been taking her antidepressant for many months. She describes having all the neurovegetative symptoms of depression and to not eating or drinking for weeks. When questioned about this, she admits to pur- posely "starving myself" as she believes that God is punishing her for directly causing the terrorist attacks of September 11, 2001. In fact, she has been "following God's instructions," which tell her to kill herself in atonement. After medical stabilization, which of the following treatments would be the most appropriate for this patient? A. Antidepressant only B. Antipsychotic only C. Electroconvulsive therapy only D. Psychotherapy only E. Psychotherapy plus antidepressant

C. This woman's history and presentation are most consistent with major depression, severe, with psychotic features. The treatment of choice for this illness is either ECT or an antidepressant/antipsychotic combination. Given the medical urgency in this case and that medication would take several weeks for significant efficacy, ECT would be preferable. Neither antidepressants nor antipsychotics alone would be as efficacious in an episode of psychotic depres- sion, and psychotherapy with or without medication would not be appropriate in someone with a severe depression as in this case.

Which of the following laboratory abnormalities would most likely be found in the patient in Question 29.2? A. Hypermagnesemia B. Hypoamylasemia C. Hypochloremic-hypokalemic alkalosis D. Elevated thyroid indices E. Hypercholesterolemia

C.Laboratoryabnormalitiesfoundinindividualswithbulimianervosademonstrate hypochloremic-hypokalemic alkalosis resulting from repetitive emesis. Hyperam- ylasemia and hypomagnesemia are also not uncommonly seen in such patients. Various electrolyte imbalances can occur as a result of frequent laxative abuse. Thyroid abnormalities are not common in individuals with bulimia nervosa.

Which symptom is more likely to be found in a patient with social phobia- related panic attacks than in a patient with a primary panic disorder? A. Paresthesias B. Derealization C. Shortness of breath D. Anticipation of an upcoming event E. Palpitations

D. Panic attacks can occur in both patients with social phobia and patients with panic disorder. In patients with social phobia, however, these panic attacks occur linked to specific social situations and can be expected to occur there. In panic disorder, however, the attacks may occur at any time.

A 63-year-old man presents to the emergency department with complaints of anxiety. He describes a long history of daily, heavy alcohol use, and 2 days ago "quit cold turkey." He appears visibly tremulous, flushed, and diaphoretic. His temperature, blood pressure, and pulse rate are elevated. The results of his physical examination are otherwise unremarkable, but his laboratory tests demonstrate low serum albumin and low protein levels, as well as an elevated prothrombin time/partial prothrombin time value. He is admitted to the med- ical service for alcohol detoxification. Which of the following medications would be most appropriate in treating this patient? A. Alprazolam B. Chlordiazepoxide C. Diazepam D. Lorazepam E. Clonazepam

D. Although all these medications are benzodiazepines, only lorazepam is metabolized solely by glucuronidation, which is not as dependent on liver functioning. The metabolism of the other benzodiazepines is much more dependent on liver function. In this patient (who has evidence of poor liver function), using high doses of medications that are dependent on liver func- tion for their degradation could result in excessive drug levels in the blood of an overly sedated patient.

Which of the following would be the most effective approach for the patient in Question 57.1? A. Confrontation about intentionally producing symptoms B. Explaining that the symptoms are not real C. Reassurance that a neurologic cause will be found D. Suggestion that symptoms will improve with time E. Suggestion that the family begins therapy

D. Although the deficits often remit spontaneously, suggesting that they will improve can facilitate the process. These patients do not intentionally pro- duce their symptoms, and explaining that their deficits are not real may aggra- vate the situation and worsen their problems. Whereas reassurance about their likely improvement is appropriate, implying that their symptoms are caused by a neurologic illness (assuming that this has been ruled out) may serve only to reinforce their defensive use of a physical illness to express their psychological problems.

A 71-year-old woman with a history of early Alzheimer disease is hospital- ized with pneumonia. During the course of the hospitalization, her family and primary physician notice a distinct worsening of her memory and alertness. Which of the following procedures would be the most sensitive in establishing a diagnosis of delirium? A. Chest radiograph B. Computerized tomography scan of the brain C. Electrocardiogram D. Electroencephalogram (EEG) E. CBC with differential

D. Although the other studies are all helpful in determining the etiology of delirium, only an EEG is sensitive in diagnosing this disorder. In almost all cases of delirium, an EEG shows generalized slowing. In cases where alcohol or sedative-hypnotic withdrawal is causing delirium, an EEG can show fast low- voltage activity. In hepatic encephalopathy, an EEG characteristically displays triphasic delta waves. Electroencephalogram findings typically remain normal early in the course of Alzheimer disease.

Atomoxetine is a relatively new drug used for the treatment of ADHD. Which of the following represents the advantage of using atomoxetine over Ritalin? A. Atomoxetine has a shorter half-life. B. Atomoxetine is available in a generic form that is less costly than Ritalin. C. Atomoxetine appears to have less of a potential for abuse than does Ritalin. D. Atomoxetine's effects begin working immediately to reduce symptoms of ADHD. E. Atomoxetine can be taken on an empty stomach.

D. Atomoxetine will not promote the tics. Methylphenidate and amphet- amine salts will promote the tics. Buproprion can stimulate dopamine and worsen tics.

A 40-year-old man with schizoaffective disorder has been hospitalized in an inpatient psychiatry unit for the third time in the last 5 years. During each episode, he becomes noncompliant in taking his medications, develops manic symptoms and auditory hallucinations, and then becomes violent. In the inpatient unit, he physically threatens other patients and staff and is generally agitated. He is put in isolation to help quiet him. The patient is prescribed a mood stabilizer. Which of the following medications might also help relieve this patient's acute agitation? A. Buspirone B. Fluoxetine C. Chloral hydrate D. Risperidone E. Benztropine

D. Atypical neuroleptics such as risperidone or quetiapine are shown to be effective in managing manic symptoms especially in the acute states while mood stabilizers reach therapeutic levels.

A 55-year-old, unresponsive woman is brought to the emergency department after an apparent suicide attempt. Earlier that day, she refilled her monthly prescription for a benzodiazepine, which she had been prescribed for panic disorder. The empty pill bottle was found on her nightstand by the paramed- ics. Concurrent ingestion of which of the following substances is most likely to worsen the prognosis of her overdose? A. Cannabis B. Cocaine C. Citalopram D. Alcohol E. Lysergic acid diethylamide (LSD)

D. Benzodiazepines are rarely lethal in overdose by themselves. However, when taken with other sedative-hypnotic drugs, especially alcohol, the effects of their potentiation can be lethal.

A 50-year-old homeless veteran is brought to the emergency department by the police for disruptive behavior. On mental status examination, he is clearly euphoric but also psychomotor agitated and somewhat paranoid; he says he "feels fantastic" but is wary of answering any questions, becoming irritated quickly. On physical examination, the patient exhibits a moderately elevated blood pressure and pulse rate. He is most likely intoxicated with which of the following substances? A. Alcohol B. Barbiturates C. Benzodiazepines D. Cocaine E. Opiates

D. Cocaine intoxication (or other stimulants like amphetamine) can present with euphoria, irritability, anxiety, psychotic symptoms such as paranoia, as well as with elevated vital signs. Conversely, intoxication with alcohol, barbi- turates, benzodiazepines, and opiates generally causes depression, somnolence, and depressed vital signs.

A 42-year-old woman undergoing psychotherapy storms into her therapist's office for her session and angrily accuses the therapist of "trying to undermine her intelligence." After a discussion with the therapist, it becomes clear that it is the patient who is second-guessing herself, thereby "undermining" her own intelligence. Which of the following defense mechanisms is this patient using? A. Denial B. Identification with the aggressor C. Intellectualization D. Projection E. Reaction formation

D. Projection is a defense mechanism by which individuals deal with conflict by falsely attributing to another their own unacceptable feelings, impulses, or thoughts. Blaming others for their own sentiments and actions directs the focus away from the person doing the accusing. For example, a patient who is angry with his therapist suddenly starts accusing the therapist of being angry with him.

A 76-year-old woman with a history of major depressive disorder, recurrent, presents to her family practice physician with a history of 4 months of increas- ing depression, associated with terminal insomnia, decreased appetite, 15 lb weight loss, fatigue, difficulty concentrating, and feelings of helplessness. She denies suicidal ideation, hallucinations, or delusions. She feels these symptoms are similar to those she had in the past. Her last episode was 30 years ago, when she was successfully treated with nortriptyline, but she has not taken any psy- chotropics since that time. She has hypertension, diabetes, and hyperlipidemia, but she has no other physical complaints. Physical examination and laboratory studies are unremarkable. After a great deal of discussion, she agrees to restart nortriptyline. When compared with the treatment of her last episode of major depression, which of the following statements is most accurate regarding medi- cation management of this episode? A. She will have fewer side effects. B. She will require higher doses. C. She will require lower blood levels. D. There is the potential for more drug-drug interactions. E. There will be a higher chance of treatment resistance.

D. Geriatric patients are often taking multiple medications and therefore have a higher likelihood of being affected by drug-drug interactions. They are also more likely to experience significant side effects from medications. Although the drug levels required to achieve efficacy are similar to those required in younger patients, because of the decreased clearance and metabolism seen in older patients, lower doses are required to reach the same levels. Efficacy of antidepressants remains similar in elderly patients.

An MRI of the brain is performed for the patient in Question 23.1. It demon- strates numerous small infarcts throughout the deep white matter structures. Which of the following best describes the likely progression of his illness? A. No change B. Gradual improvement C. Gradual worsening D. Stepwise deterioration E. Rapid progression

D. Given his history of hypertension and lacunar white-matter infarcts, his diagnosis is most likely vascular (multi-infarct) dementia. Vascular dementia characteristically demonstrates a stepwise deterioration in cognitive function- ing corresponding to either small ischemic events or emboli causing strokes. Alzheimer disease progresses gradually over many years, whereas dementias caused by head trauma or anoxia are generally sudden in onset and have a subsequently stable course. Dementias with certain infectious etiologies, such as Creutzfeldt-Jakob disease, have a fairly rapid rate of progression.

A 42-year-old man returns to his internist for the fourth time in 5 months with the same symptoms of intermittent numbness of his fingers and indiges- tion. Although his medical workup has been unremarkable, this has failed to reassure him. He is now concerned that he has celiac disease and requests a gastrointestinal (GI) consultation. Which of the following is the most likely diagnosis? A. Conversion disorder B. Factitious disorder C. Malingering D. Hypochondriasis E. Body dysmorphic disorder

D. Hypochondriasis is characterized as a person's preoccupation with the fear of contracting or the belief of having a serious disease. This fear stems from a misinterpretation of physical symptoms or functions. Unlike conversion disor- der, hypochondriasis is not confined to one or more sensory or motor deficits. Since the patient is not consciously faking symptoms to maintain a sick role, this would not be considered a factitious disorder. If the patient were malinger- ing there would be an obvious secondary gain. Patients with body dysmorphic disorder have a preoccupation with an imagined defect in appearance.

Despite a course of psychotherapy, the patient in Questions 21.1 and 21.2 continues to suffer from recurrent nightmares, flashbacks, hypervigilance, and emotional numbing. Which of the following medications is most likely to be helpful as monotherapy in this patient? A. Buspirone B. Risperidone C. Alprazolam D. Prazosin E. Valproic acid

D. Individuals with PTSD often respond to SSRIs or SNRIs. Addition- ally, alpha-1 antagonists such as prazosin have demonstrated efficacy in significantly reducing the symptom clusters of PTSD. However, buspirone, second-generation ("atypical") antipsychotics (such as risperidone), ben- zodiazepines, and mood stabilizers (such as valproic acid) are not recom- mended as monotherapy for the treatment of PTSD. Although alprazolam might assist in decreasing the patient's general anxiety, the incidence of substance abuse is high among patients with PTSD; thus, addictive medica- tions should be avoided in these individuals.

A woman with schizoid personality disorder was involved in a motor vehicle accident in which she was rear-ended by another car. The driver of the other car refused to take responsibility for the accident and hired a lawyer to provide his defense. The woman spends hours each day thinking about the specifics of the accident, including such details as the color of the cars involved and what each party to the accident was wearing. Which of the following defense mechanisms, common to patients with schizoid personality disorder, is the woman using? A. Sublimation B. Undoing C. Projection D. Intellectualization E. Introjection

D. Intellectualization is characterized by rehashing events over and over.

A 33-year-old woman with bipolar disorder is 22 weeks pregnant. She has been taking valproic acid for her symptoms. Which of the following is the most likely abnormality that might be found on an ultrasound examination, due to the effects of the mood stabilizer? A. Fetal abdominal wall defect B. Fetal microcephaly C. Fetal renal dysplasia D. Fetal spina bifida E. Fetal tetralogy of Fallot

D. Maternal use of valproic acid is associated with a 1% to 2% risk of fetal neural tube defects such as spina bifida.

Which of the following medications would be the best choice for treating nonpsychotic mania in a 10-year-old boy? A. Isotretinoin (Accutane) B. Beclomethasone C. Lithium D. Divalproex E. Risperidone

D. Mood stabilizers are used to treat bipolar disorder. Lithium is the only medi- cation that has received FDA approval for the treatment of bipolar disorder in youth aged more than 12 years, but divalproex has a longer safety profile in young individuals given its long history of use for seizures.

Given the loss of a spouse suffered by the patient in Question 23.1, a depressive disorder (pseudodementia) as a cause of his memory problems is considered in his differential diagnosis. How would this patient be predicted to perform on cognitive testing if he has a dementia rather than a depressive illness? A. Reduced effort with poor insight B. Reduced effort with good insight C. No effort with poor insight D. Better effort with poor insight E. Better effort with good insight

D. On cognitive testing, patients with dementia generally put forth consider- able effort, display poor insight into their deficits, and minimize their prob- lems. In contrast, those with depressive disorders often are apathetic, make little effort, but complain about their memory problems.

A 35-year-old woman with histrionic personality disorder has seen her psy- chotherapist once a week for the past year. During a session, the therapist tells the patient that he is going to be on vacation the following 2 weeks. When he returns from the vacation, the patient tells him that she felt he abandoned her and says, "You didn't even bother to tell me that you would be away." This lapse in memory can best be described as which defense mechanism common to patients with histrionic personality disorder? A. Sublimation B. Splitting C. Undoing D. Repression E. Displacement

D. Repression is a common defense mechanism in patients with histrionic personality disorder.

A 45-year-old woman with alcohol dependence in full remission is referred to your practice. She has a history of GAD and was recently prescribed a ben- zodiazepine by her internist. Which of the following is the most appropriate treatment for her? A. Continue the benzodiazepine and an SSRI. B. Buspirone monotherapy. C. Continue benzodiazepine. D. Selective serotonin reuptake inhibitor monotherapy. E. Switch to a long-acting benzodiazepine.

D. SSRIs are first-line treatment for GAD. Potentially addictive medications such as benzodiazepines should be avoided in patients with a history of sub- stance dependence. Buspirone is not as effective in patients already exposed to benzodiazepines.

A 29-year-old woman presents to establish care. She complains of a 6-month history of headaches, fatigue, lower extremity tingling, decreased libido, diffuse myalgias and arthralgias, and depression. She and her husband and recently separated and she has taken a leave of absence from work because of her physi- cal problems. Three years ago she had an episode of "lightning pains" down her arms that resolved. Her past medical history is otherwise unremarkable and she has no history of psychiatric illnesses. She was adopted and has no knowledge of her family medical history. While talking to you, she becomes tearful. What is the most appropriate next step? A. Refer the patient for counseling. B. Start an antidepressant. C. Encourage her to see the connection between her physical symptoms and psychological distress. D. Complete a detailed history, physical examination, and order appropriate blood work. E. Schedule her for a return appointment within the next 2 weeks.

D. Somatization disorder generally presents with nonspecific symptoms. The most important initial step is to rule out medical causes.

Which of the following scenarios is most consistent with factitious disorder? A. Feigning psychosis to avoid criminal charges B. Lying about back pain to receive time off from work C. Pseudoseizures in the context of a family conflict D. Placing feces in urine to receive treatment for a urinary tract infection E. Recurrent fears of having a serious illness

D. The hallmark of factitious disorder is intentional feigning of a physical or psychiatric illness in order to assume the sick role. Examples include injecting oneself with insulin to create hypoglycemia, taking anticoagulants to fake a bleeding disorder, and contaminating urine samples with feces to simulate a urinary tract infection. Lying about back pain in order to avoid work or feigning psychosis to avoid criminal charges is an example of malingering. Pseudoseizures are an example of a conversion disorder. Fear of having a seri- ous disease caused by misinterpretation of bodily sensations is characteristic of hypochondriasis.

A 48-year-old woman is brought to the emergency department. She is unre- sponsive to questions, stumbles around the room, and is agitated. On physical examination, you notice that she smells of alcohol, and she is not cooperative during the remainder of the examination. Administration of what medicine would be the most appropriate initial treatment? A. Benzodiazepine B. Disulfiram C. Glucose D. Thiamine E. An antipsychotic agent

D. The most appropriate treatment is administration of thiamine. This patient presents with Wernicke encephalopathy, characterized by the triad of delir- ium, ataxia, and ophthalmoplegia. Thiamine must be given prior to glucose in patients suspected of having this disorder.

Which of the following strategies by the primary care doctor would be the most effective in treating the patient in Question 24.3? A. Antianxiety medication B. Extensive medical workups to provide reassurance C. Referral for psychotherapy D. Regularly scheduled appointments with reassurance E. An antipsychotic medication

D. The most effective strategy for treating individuals with hypochondriasis is to schedule regular appointments. In this way, any physical complaints are addressed, and reassurance is provided, albeit temporarily. This approach also minimizes both doctor shopping and unnecessary testing. Treatment with an antianxiety (or antidepressant) agent is not helpful in hypochondriasis, unless a comorbid anxiety (or depressive) disorder is present. Because indi- viduals with this disorder are fearful of having a medical illness, they usually resist seeing a psychiatrist.

A 28-year-old man describes a persistent fear of speaking in public. Although he does not have difficulty with one-on-one situations, when giving a lec- ture he becomes extremely anxious, worrying that he will be humiliated. He relates one episode in which he was forced to speak at the last minute, which resulted in his experiencing panic, shaking, abdominal cramps, and a fear that he would defecate on himself. Because of this problem, he has been held back from promotion at his place of business. Which of the following is the most likely diagnosis? A. Generalized anxiety disorder B. Panic disorder with agoraphobia C. Panic disorder without agoraphobia D. Social phobia E. Specific phobia

D. The most likely diagnosis for this man is social phobia. Although he suffers from panic attacks, they are not unprovoked as in panic disorder because they occur in response to public speaking. His fear is not of having further attacks but rather of being embarrassed or humiliated. For generalized anxiety disorder the anxiety and worry is not confined to being embarrassed as in social phobia, but rather involves excessive anxiety and worry about a number of events or activities. Specific phobia is characterized by an excessive fear that is cued by the presence or anticipation of a specific object or situation (eg, flying, animals) rather than social or performance situations.

An 18-year-old man presents with 3days of an irritable mood, decreased sleeping, talkativeness, increased energy, and distractibility. He has no personal or family psychiatric history and no current medical problems. His mental status examina- tion is remarkable for psychomotor agitation and an irritable affect. He is paranoid but denies delusions or hallucinations. His physical examination is notable for a slightly elevated pulse rate and blood pressure as well as markedly dilated pupils bilaterally. The result of his urine toxicology screen is positive for cocaine. A. Bipolar disorder, manic B. Major depression C. Mood disorder due to a general medical condition D. Substance-induced mood disorder E. Adjustment disorder with depressed mood

D. The most likely diagnosis for this man is substance (cocaine)-induced mood disorder. Although he presents with classic manic symptoms (irritable mood, decreased sleep, etc), he has no psychiatric or family history of mood disorder. His physical examination reveals several findings not necessarily consistent with mania, namely, elevated vital signs (pulse rate and blood pressure) and dilated pupils. The important factor in this case is his obvious cocaine use, which can produce symptoms mimicking those of acute mania.

A 36-year-old man with a past history of a major depressive episode is brought into the emergency room by the police after stopping traffic on the highway proclaiming that he is "the Messiah." His wife is contacted who states that he has been walking throughout the house all night for the last 4 nights, talking "nonstop," and starting many home repair projects that remain unfinished. She believes that he is taking sertraline for his depression and propranolol for high blood pressure. His blood alcohol level is less than 10, and his urine toxicology screen is negative. A. Bipolar disorder, manic B. Major depression C. Mood disorder due to a general medical condition D. Substance-induced mood disorder E. Adjustment disorder with depressed mood

D. The most likely diagnosis for this man is substance-induced mood disor- der, in other words mania caused by antidepressant treatment. He has clas- sic symptoms and signs of mania, such as decreased need for sleep, talkative- ness, increased activity, risky behavior, and delusions of grandeur. He also has a history of a major depressive episode and has been taking an antidepressant which likely has caused a switch into his current manic episode. According to the DSM-IV-TR, manic episodes caused by antidepressant treatment are consid- ered substance induced and do not count toward a diagnosis of bipolar disorder. Although he is taking a beta-blocker for hypertension, this would be likely to cause a depressive episode rather than a manic episode.

A 42-year-old woman describes a 20-year history of numerous physical com- plaints, including joint pain, dysuria, headaches, chest pain, nausea, vomiting, irregular menses, and double vision. Although they do not all occur at the same time, she has been suffering from one or more of these problems through- out her adult life. Many workups have been done for her, and she has under- gone repeated hospitalizations, but no specific cause has yet been found. She is extremely anxious and has become significantly disabled as a result. Which of the following is the most likely diagnosis? A. Body dysmorphic disorder B. Hypochondriasis C. Pain disorder D. Somatization disorder E. Conversion disorder

D. The most likely diagnosis for this woman is somatization disorder. She pres- ents with numerous somatic complaints, related to several body areas, which are not fully explained by a medical cause. The focus is on the symptoms them- selves, not on a perceived physical defect (as in body dysmorphic disorder), on the fear of having a specific disease (as in hypochondriasis), or on symptoms of pain (as in pain disorder).

A 43-year-old divorced female veteran with schizophrenia is being followed in an outpatient community mental health clinic after being discharged from the hospital. Her medications have subsequently been increased to risperidone 3 mg in the morning and 4 mg in the evening. She has some paranoia and ideas of reference, but she denies auditory or visual hallucinations. Her mental status examination is significant for moderate psychomotor slowing, with little spontaneous speech, but with a coarse tremor of her hands bilaterally. Her stated mood is "fine," although her affect appears blunted, with little expres- sion. Her gait is wide based and shuffling. A. Acute dystonic reaction B. Akathisia C. Neuroleptic malignant syndrome D. Parkinsonism E. Tardive dyskinesia

D. The patient is a middle-aged female veteran with chronic psychotic symp- toms, just released from the hospital with an increase in her risperidone dose. She now demonstrates bradykinesia, shuffling gait, masked facies, and a coarse tremor, all consistent with antipsychotic-induced parkinsonism. Risk factors for the development of parkinsonism include female gender and an older age.

For the patient described in Question 20.3, which of the following treatments might best be used by the psychiatrist? A. Lithium B. Interpersonal psychotherapy C. Buspirone D. Cognitive-behavioral therapy (CBT)-evoked response prevention E. Parent assertiveness training

D. The standard pharmacologic approach to the treatment of OCD is to prescribe a selective serotonin reuptake inhibitor (SSRI) or clomipramine, although these are not answer choices. The best psychotherapeutic choice would involve gradually exposing the patient to the anxiety-provoking circum- stance and teaching her how to manage that anxiety through CBT techniques.

For a diagnosis of somatization disorder, which of the following criteria must be met? A. External motivation for symptoms (avoid work or financial gain). B. Patient is without significant impairment. C. Symptoms are intentionally feigned or produced. D. Symptoms begin prior to age 30. E. Symptoms last less than 6 months.

D. The symptoms must occur prior to age 30 and be present for several years.

You are consulted to evaluate a 45-year-old, married woman who was admitted to the surgical service 2 days ago for an appendectomy. The procedure went well, but she was found to be tearful, stating, "I wish I were dead." On obtaining further history, she is quite cooperative and talkative. She is questioned about her earlier comments, and she states that she "wanted attention, I guess." She is upset that her husband is not with her in the hospital; she has "never been away from him" for this long since they started dating when the patient was 16 years old. She feels helpless and is having a difficult time being active in her care. She feels overwhelmed regarding her postsurgical and discharge instruc- tions, and the nursing staff has become frustrated with her constant "need for reassurance." Although at times she is tearful during the interview, she denies prior or recent pervasive depressive or neurovegetative symptoms and is not actively suicidal. Which of the following is the most appropriate approach to this patient? A. Encourage her to learn more about her surgery and become more proac- tive in her care. B. Persuade her to become less dependent on her husband. C. Persuade her husband to come spend more time with her at the hospital. D. Spend regular, short periods of time with her to discuss discharge planning and aftercare. E. Transfer her to the psychiatric unit.

D. This patient displays characteristics consistent with dependent personal- ity disorder. The most effective approach in dealing with a patient with this disorder is to respect her need for attachment and schedule limited but regular appointments with her. Individuals with this illness wish to be taken care of, and therefore will not be proactive in their care; in circumstances such as these, it is helpful for the physician to be more active. Encouraging the patient to be less dependent in her primary relationship is not only not helpful, but can be damaging as she can feel rejected and become more regressed, upset, and helpless. Encouraging the husband to spend more time at the hospital only continues the cycle of "being taken care of" and does not facilitate the patient's involvement in postoperative care.

An 18-year-old man is brought to the emergency department by his friends after he started a fight with one of them and subsequently was knocked unconscious. On awakening, the patient states that he was fighting for the "freedom of the world" and that he was told by "the voices in his head" that his friends held the key to winning the battle. The patient is irritable and restless and paces around in the emergency department. He is unable to sit still for the interview and pre- fers to stand up, keeping his back to the door as he speaks with the physician. The patient's friends state that he has been withdrawing from them and his schoolwork over the past 18 months and that they think he has become "odd." They report that his mood has become irritable only over the past several days. They say that he hoards random scraps of paper in his room and that his grades have dropped dramatically over the past 6 months. The results of a toxicology screening are negative, as are those of a physical examination, although the examination was limited because of poor cooperation on the part of the patient. Which of the following is the most likely diagnosis? A. Bipolar disorder, manic B. Mood disorder secondary to a general medical condition C. Schizoaffective disorder D. Schizophrenia E. Substance-induced mood disorder

D. This patient is currently experiencing a frank episode of psychosis, which appears to most recently have escalated with the appearance of paranoia (standing with his back to the door as he speaks with the physician) and extreme irritability, which could be confused with manic symptoms. How- ever, he has a history of a disturbance in premorbid functioning that predates the irritability by 18 months. He is also in the prime age range for new-onset schizophrenia, making this the more likely diagnosis.

A 47-year-old man is admitted to a psychiatric unit for depression with suicidal ideation and detoxification. He has a long history of dependence upon both alcohol and cocaine. Which of the following signs is most characteristic of early alcohol withdrawal? A. Decreased blood pressure B. Hypersomnia C. Persistent hallucinations D. Tremor E. Increased appetite

D. Tremor is the most characteristic sign of alcohol withdrawal. Vital signs are elevated in alcohol withdrawal because of autonomic hyperactivity. Patients generally have insomnia as a result, not hypersomnia. Hallucinations associ- ated with alcohol withdrawal usually resolve within a week, while those occur- ring in delirium tremens usually resolve with the delirium. Cocaine withdrawal more typically involves hypersomnia and hyperphagia.

A 39-year-old man is evaluated by mental health services in prison. He has a history of multiple arrests as both an adult and as a juvenile. After several interviews, a diagnosis of antisocial personality disorder is confirmed. He has a history of multiple psychiatric hospitalizations after suicide attempts and was in special education programming as a child. Which psychiatric diagnosis is most likely to have occurred comorbidly in such an individual? A. Attention-deficit/hyperactivity disorder B. Cocaine dependence C. Traumatic brain injury D. Major depression E. Conduct disorder

E. Although all the diagnoses listed are frequently comorbid, evidence of a diagnosis of conduct disorder with onset before age 15 is required for a diagnosis of antisocial personality disorder.

A 22-year-old woman is referred to your office by her family physician for evaluation of "depression." Her primary care doctor is unsure whether she is suffering from dysthymic disorder or a major depressive disorder. Which of the following characteristics is more consistent with dysthymic disorder versus major depression? A. Episodic course. B. Numerous neurovegetative symptoms. C. Presence of psychotic symptoms. D. Severe impairment in functioning. E. Symptoms are of a low intensity.

E. Although the distinction between dysthymic disorder and major depres- sive disorder can sometimes be challenging (especially if the major depres- sive illness is chronic and/or recurrent), patients with dysthymic disorder tend to have an earlier onset of lower-intensity symptoms, a more chronic course, fewer neurovegetative symptoms, lack of psychosis, and less severe psychoso- cial or occupational impairment when compared to individuals with major depression.

A diagnosis of GAD is made for a 41-year-old female accountant who has had moderate but distressing symptoms for several years. She has strong negative feelings about taking any psychotropic medication, even after being educated about pharmacologic treatment options. Which of the following would be the most efficacious treatment option? A. Cognitive-behavioral therapy only B. Psychoanalysis only C. Psychoeducation only D. Supportive psychotherapy only E. Cognitive-behavioral therapy with an SSRI, venlafaxine, or buspirone

E. Because the patient's symptoms are moderate and she feels so negatively about medication, the best option for psychotherapy is CBT, the most-studied evidence-based psychotherapy. Other interventions such as insight-oriented psychotherapy and supportive psychotherapy are less likely to be effective. This should be combined with a nonaddictive medication for anxiety.

A 25-year-old man comes to the hospital with symptoms of gastrointestinal distress, muscle aches, rhinorrhea, lacrimation, and an anxious mood. He states that he "wants to kick this thing once and for all." Which of the follow- ing medications would be most helpful in ameliorating his symptoms? A. Antabuse B. Haloperidol C. Naloxone D. Lorazepam E. Clonidine

E. Clonidine can be used to help ease the withdrawal symptoms of opioid withdrawal. It is not an opioid and does not have any addictive properties. However, the withdrawal may not be as painless as it would be if methadone were used. Blood pressure levels should be monitored when clonidine is used. Antabuse or disulfiram is a treatment option for alcoholics. It is not used to treat acute opioid withdrawal. Naloxone is used to counter the effects of life-threatening depression of the central nervous system and respiratory system from opioid overdose (eg, heroin or methadone overdose). It would worsen this patient's withdrawal symptoms. Lorazepam, a benzodiazepine, is commonly used in the treatment of alcohol withdrawal. Haloperidol, an anti- psychotic, has no use in treating withdrawal.

A 75-year-old man is brought in by his daughter to a psychiatrist for an evalu- ation. He has become increasingly forgetful over the past year, missing engage- ments with his children and grandchildren. He is also unable to remember directions, resulting in his becoming lost when driving alone. He has no psy- chiatric history, although his wife died 14 months ago. His medical history is significant for poorly controlled hypertension. Which of the following addi- tional features is necessary in order to accurately diagnose dementia? A. Agitation B. Fluctuation in consciousness C. Radiographic findings D. Hallucinations E. Another cognitive deficit

E. For a diagnosis of dementia, one or more additional cognitive deficits must be present in addition to memory impairment. They can include aphasia, apraxia, agnosia, or a disturbance in executive functioning. Individuals with vascular dementia can demonstrate findings on a CT or MRI scan, such as lacunar infarcts or evidence of past strokes. Patients with Alzheimer disease may demonstrate generalized cortical atrophy and ventricular enlargement. Individuals with dementia usually remain alert, whereas those with delirium display a fluctuation in consciousness. While psychotic symptoms such as de- lusions and hallucinations, as well as agitation, can be seen, they are neither specific to nor necessary for the diagnosis of dementia.

Which of the following would be the most useful psychiatric treatment for the patient in Question 10.2? vA. Antianxiety medication B. Antidepressant medication C. Antipsychotic medication D. Electroconvulsive therapy E. Group psychotherapy

E. Psychotherapies, such as insight-oriented, family, behavioral, and group therapy have been useful in patients with dependent personality disorder. Medi- cations for depression, anxiety, and/or psychosis, or electroconvulsive therapy (ECT), would only be indicated if the patient had a comorbid psychiatric illness, not evident in the patient in Question 10.2.

The patient in Question 22.1 is evaluated fully and determined to have dys- thymic disorder. Which of the following medications is the most appropriate first-line treatment for her? A. Desipramine B. Lithium C. Lorazepam D. Phenelzine E. Sertraline

E. SSRIs (such as sertraline), SNRIs, and bupropion have demonstrated efficacy in treating dysthymia. Although TCAs and MAOIs are also beneficial, newer antidepressants such as SSRIs or SNRIs are better tolerated and safer in overdose. Neither lithium nor lorazepam is indicated for dysthymic disorder.

A 25-year-old woman describes a lifelong history of being "scared of heights." She becomes uncomfortable when at an elevation higher than three stories and whenever traveling or shopping becomes preoccupied with knowing the heights of buildings. On finding herself at a significant distance from the ground, she has severe anxiety symptoms including trembling, lightheadedness, numbness and tingling, and a fear of dying. Which of the following is the most likely diagnosis? A. Generalized anxiety disorder B. Panic disorder with agoraphobia C. Panic disorder without agoraphobia D. Social phobia E. Specific phobia

E. Specific phobia is the most likely diagnosis for this woman. Although she has panic attacks, they are not unexpected and result from being in an elevated location. Her fears are of a particular situation (heights) rather than of having further panic attacks.

In the patient from Question 46.2, his play with tops and balls might be con- sidered a stereotyped behavior. Which of the following statements best defines stereotypical behavior? A. Interacting preferentially with other children of a specific race B. Preference for similar schedules of play every day C. Focus of play on only one aspect of a toy D. Difficulty playing with a toy creatively beyond its most obvious use E. Spinning or repetitively using a toy in a specific manner for extended periods of time

E. Spinning or repetitively using a toy in a specific manner for extended peri- ods of time is characteristic of stereotyped play. Autistic individuals often play with toys or objects in odd or eccentric ways. A typical example of stereotyped behavior in playing with toys is spinning or moving an object or toy in the same way for long periods of time.

The patient in Questions 16.1 and 16.2 is subsequently admitted to a detoxi- fication unit in the hospital. After his euphoria and paranoia resolve, he is able to give a more complete history. He describes a 5-year history of almost daily crack cocaine use, with no periods of sobriety lasting for greater than 7 to 10 days—during these periods, he felt "depressed," with an increased appetite, disrupted sleep, difficulty concentrating, and fatigue. He denies alco- hol or other drug use, and his psychiatric review of systems is otherwise nega- tive. Which of the following would be the most appropriate initial treatment for this patient? A. Antidepressant B. Dopamine antagonist C. Dopamine agonist D. Mood stabilizer E. Narcotics Anonymous

E. The most beneficial approach to cocaine addiction without additional psy- chopathology is to stress abstinence and relapse prevention. Narcotics Anony- mous meetings are easily accessible, approachable, and provide ongoing group and individual support. No medications have been consistently proven to prevent cocaine cravings or relapse. While the patient admits to depressive symptoms when abstinent from cocaine, these symptoms are quite common (and self-limiting) during withdrawal. In addition, the time period is not typi- cal of a major depressive disorder. After 4 to 6 weeks of sobriety, if depressive symptoms are still present, he should be reevaluated with consideration to begin an antidepressant.

A 44-year-old woman comes to your office for a follow-up visit. She recently received a diagnosis of major depressive disorder and began treatment with citalopram (an SSRI) 6 weeks ago. She claims to feel "happy again," without further depression, crying spells, or insomnia. Her appetite has improved, and she has been able to focus at work and enjoy time with her family. Although she experienced occasional headaches and loose stools at the beginning of her treatment, she no longer complains of any side effects. Which of the following is the most appropriate next step in her treatment? A. Consider a different class of antidepressants. B. Discontinue the citalopram. C. Increase the dose of citalopram. D. Lower the dose of citalopram. E. Maintain the current dose of citalopram.

E. The proper strategy in the management of an episode of major depres- sion that has recently remitted is to continue treatment at the same dose if it can be tolerated. Early discontinuation of medication can lead to an early relapse. A general rule of thumb is, "The dose that got you better will keep you well." A reasonable duration for continuing the medication is 6 to 9 months.

Acute stress disorder is diagnosed in a 32-year-old woman who witnessed her fiancé being shot to death in a robbery attempt. She has difficulty sleeping and feels that she is not emotionally attached to anything around her. She also has repetitive flashbacks of the event and avoids going near the location where the incident occurred. Which of the following medications might be helpful to this patient over the short term? A. Buspirone B. Paroxetine C. Risperidone D. Valproate E. Zolpidem

E. The use of a hypnotic for insomnia is likely to be helpful to this patient in the short term. Buspirone is used for those patients diagnosed with generalized anxiety disorder, while paroxetine is used in the treatment of major depression. Risperidone is an antipsychotic and valproate is used for mood stabilization of patients with bipolar disorder.

Which of the following is an associated disorder in the family histories of patients with schizotypal personality disorder? A. Alcohol dependence B. Bipolar disorder C. Hypochondriasis D. Panic disorder E. Schizophrenia

E. There is a greater association of cases of schizotypal personality disorder among biological relatives of patients with schizophrenia than among con- trols. The other disorders listed are not associated with higher rates of person- ality disorders.

Which of the following treatment options would be contraindicated among recommended treatment options for the patient in Questions 29.2 and 29.3? A. Nutritional rehabilitation B. Cognitive-behavioral psychotherapy C. Careful use of SSRIs D. Group psychotherapy E. Atypical antipsychotic medications to increase her appetite

E. There is no clinical evidence for the use of atypical antipsychotics in a patient with bulimia.

A 50-year-old woman with a diagnosis of schizoaffective disorder, bipolar type, complains of "nervous tics." She denies significant affective symptoms but complains of chronic auditory hallucinations of "whispers" without com- mands. No suicidal or homicidal ideation is present. On examination, she is noted to be sticking her tongue in and out of her mouth and to have repetitive, rhythmic movements of her hands and feet. A. Acute dystonic reaction B. Akathisia C. Neuroleptic malignant syndrome D. Parkinsonism E. Tardive dyskinesia

E. This female patient with chronic schizoaffective disorder now demon- strates choreoathetoid movements of her tongue and extremities, consistent with tardive dyskinesia. She has several risk factors for development of tardive dyskinesia, including likely long-term treatment with antipsychotics, female gender, and a mood disorder.

A 35-year-old woman is engaged in psychotherapy to address her avoidant per- sonality disorder. In particular, she is distressed by her inability to maintain a romantic relationship with a man. During the course of treatment, the therapist learns that her father was alcoholic, and was physically abusive to the patient and her mother. Which defense mechanism best describes the patient's behavior? A. Undoing B. Splitting C. Isolation of affect D. Idealization E. Displacement

E. This patient can be theorized to be using displacement to assume that all men will act as punitively toward her as her father did. Displacement and pro- jection are the two defense mechanisms most commonly utilized by patients with avoidant personality disorders.


Ensembles d'études connexes

Chapter 7. Relationship Development

View Set

Programming Logic and Design: Ch 1-5

View Set

Nutrition Concepts and Controversies, Ch. 4

View Set

Hawkes Learning Business Statistics 4.3

View Set

SKILL SET ATI ( MOBILITY, INFECTION, BOWEL ELIMINATION, OSTOMY CARE)

View Set

AP Macroeconomics Unit 2 Progress Check: MCQ

View Set